Barron's SAT, 26th edition (2012)

Part 2. PINPOINT YOUR TROUBLE SPOTS

A Diagnostic Test

123

• Diagnostic Test

• Answer Key

• Self-Evaluation

• Answer Explanations



The diagnostic test in this chapter is a multipurpose tool.

• Please write all answers on separate sheets of paper. The directions are for reference only. Directions are similar to the real exam and not for this e-book.

• First, it will help you identify your problem areas and skills. Take the test and evaluate your results, following the charts provided. You will discover your strengths and weaknesses, and you will know what to study.

• Second, this test will help you design a study plan that’s right for you. Use the information you get from your result to tailor a study plan to fit your particular needs. If you need extra time on a certain topic, build time in. You are in charge of your study program—make it work for you.

• Third, this test is your introduction to the format and content of the SAT. There is nothing like working your way through actual SAT-type questions for 3 hours and 20 minutes to teach you how much stamina you need and how much speed.

• Finally, this test is your chance to learn how to profit from your mistakes. It will expose you to the sorts of traps the test-makers set for you and the sorts of shortcuts that you should take. Read the answer explanation for every question you miss or omit. You’ll be amazed to see how much you’ll learn.

• The diagnostic test contains hyperlinks that allow you to click between questions and their answers.

As shown in the chart on, an actual SAT has ten sections and takes a little more than 4 hours to complete, including two 10-minute breaks. One of those ten sections is an experimental section that is not included in your scores. This diagnostic test, as well as all the model tests in this book, has only nine sections—it does not have the experimental section—and so it takes about 3 hours and 45 minutes to complete: 3 hours and 20 minutes of actual time working on the test, plus six 1-minute breaks between sections and two 10-minute breaks, after Sections 3 and 7.

You are about to take a diagnostic test that can change the way you do on the SAT. You have 3 hours and 45 minutes to get through the nine sections (numbered 1–4 and 6–10; there is no Section 5), with breaks. Make every minute count.

Answer Sheet—Diagnostic Test

Section 1                ESSAY

img

If a section has fewer questions than answer spaces, leave the extra spaces blank.

* The answer sheets are for reference only.

Section 2

img

Section 3

img

Section 4

img

Section 6

img

Section 7

img

img

Section 8

img

Section 9

img

Section 10

img

Diagnostic Test

Section 1

Time—25 Minutes

ESSAY


The excerpt appearing below makes a point about a particular topic. Read the passage carefully, and think about the assignment that follows.

Since the invention of television, the medium has had its ups and downs. At first, television watching was a communal affair; the first television set owners in a neighborhood would proudly invite the neighbors in to view the marvelous box. In time, however, television came to have an isolating effect on viewers; as the painter Andy Warhol once said, “When I got my first television set, I stopped caring so much about having close relationships.”

ASSIGNMENT: What are your thoughts on the idea that television has turned out to isolate people instead of bringing them together? Compose an essay in which you express your views on this topic. Your essay may support, refute, or qualify the views expressed in the excerpt. What you write, however, must be relevant to the topic under discussion. Additionally, you must support your viewpoint, explaining your reasoning and providing examples based on your studies and/or experience.

Section 2

Time—25 Minutes 24 Questions

Select the best answer to each of the following questions; then blacken the appropriate space on your answer sheet.

Each of the following sentences contains one or two blanks; each blank indicates that a word or set of words has been left out. Below the sentence are five words or phrases, lettered A through E. Select the word or set of words that best completes the sentence.

Example:

Fame is ----; today’s rising star is all too soon tomorrow’s washed-up has-been.

(A) rewarding

(B) gradual

(C) essential

(D) spontaneous

(E) transitory

img

  1. Because of their frequent disarray, confusion, and loss of memory, those hit by lightning while alone are sometimes ---- victims of assault.

(A) mistaken for

(B) attracted to

(C) unaware of

(D) avoided by

(E) useful to

  2. Having published more than three hundred books in less than fifty years, science fiction writer Isaac Asimov may well be the most ---- author of our day.

(A) fastidious

(B) insecure

(C) outmoded

(D) prolific

(E) indigenous

  3. Because his time was limited, Weng decided to read the ---- novel War and Peace in ---- edition.

(A) wordy..an unedited

(B) lengthy..an abridged

(C) famous..a modern

(D) romantic..an autographed

(E) popular..a complete

  4. In giving a speech, the speaker’s goal is to communicate ideas clearly and ----, so that the audience will be in no ---- about the meaning of the speech.

(A) effectively..haste

(B) indirectly..distress

(C) vigorously..discomfort

(D) unambiguously..confusion

(E) tactfully..suspense

  5. Although gregarious by nature, Lisa became quiet and ---- after she was unexpectedly laid off from work.

(A) autonomous

(B) susceptible

(C) assertive

(D) withdrawn

(E) composed

  6. The increasingly popular leader of America’s second largest tribe, Cherokee Chief Wilma Mankiller, has ---- the myth that only males can be leaders in American Indian government.

(A) shattered

(B) perpetuated

(C) exaggerated

(D) confirmed

(E) venerated

  7. The commission of inquiry censured the senator for his ---- expenditure of public funds, which they found to be --.

(A) flagrant..cursory

(B) improper..vindicated

(C) lavish..unjustifiable

(D) judicious..blameworthy

(E) arbitrary..critical

  8. Despite their ---- of Twain’s Huckleberry Finn for its stereotyped portrait of the slave Jim, even the novel’s ---- agreed that it is a masterpiece of American prose.

(A) admiration..critics

(B) denunciation..supporters

(C) criticism..detractors

(D) defense..censors

(E) praise..advocates

Read each of the passages below, and then answer the questions that follow the passage. The correct response may be stated outright or merely suggested in the passage.

Questions 9 and 10 are based on the following passage.

Consider the humble jellyfish. Headless, spineless,
without a heart or brain, it has such a simple
exterior that it seems the most primitive of
creatures. Unlike its sessile (attached to a surface,

Line (5) as an oyster is attached to its shell) relatives whose
stalks cling to seaweed or tropical coral reefs, the
free-swimming jellyfish, or medusa, drifts along
the ocean shore, propelling itself by pulsing, muscular
contractions of its bell-shaped body. Yet

(10) beneath the simple surface of this aimlessly drifting,
supposedly primitive creature is an unusually
sophisticated set of genes, as recent studies of the
invertebrate animal phylum Cnidaria (pronounced
nih-DARE-ee-uh) reveal.

  9. Which assertion about jellyfish is supported by the passage?

(A) They move at a rapid rate.

(B) They are cowardly.

(C) They lack mobility.

(D) They have a certain degree of intelligence.

(E) They are unexpectedly complex.

10. The last sentence of the passage serves primarily to

(A) explain the origin of a term

(B) contradict an assumption

(C) provide an example

(D) cite a well-known fact

(E) describe a process

Questions 11 and 12 are based on the following passage.

The passage below is excerpted from Somerset Maugham’s The Moon and Sixpence, first published in 1919.

The faculty for myth is innate in the human
race. It seizes with avidity upon any incidents,
surprising or mysterious, in the career of those
who have at all distinguished themselves from

Line (5) their fellows, and invents a legend. It is the
protest of romance against the commonplace of
life. The incidents of the legend become the
hero’s surest passport to immortality. The ironic
philosopher reflects with a smile that Sir Walter

(10) Raleigh is more safely enshrined in the memory
of mankind because he set his cloak for the
Virgin Queen to walk on than because he carried
the English name to undiscovered countries.

11. In line 1, “faculty” most nearly means

(A) capacity

(B) distinction

(C) authority

(D) teaching staff

(E) branch of learning

12. In lines 8–13, the author mentions Sir Walter Raleigh primarily to

(A) demonstrate the importance of Raleigh’s voyages of discovery

(B) mock Raleigh’s behavior in casting down his cloak to protect the queen’s feet from the mud

(C) illustrate how legendary events outshine historical achievements in the public’s mind

(D) distinguish between Raleigh the courtier and Raleigh the seafarer

(E) remind us that historical figures may act in idiosyncratic ways

Questions 13–24 are based on the following passage.

The passage below is excerpted from the introduction to Bury My Heart at Wounded Knee, written in 1970 by the Native American historian Dee Brown.

Since the exploratory journey of Lewis and
Clark to the Pacific Coast early in the nineteenth
century, the number of published accounts
describing the “opening” of the American West

Line (5) has risen into the thousands. The greatest concentration
of recorded experience and observation
came out of the thirty-year span between 1860
and 1890—the period covered by this book. It
was an incredible era of violence, greed, audacity,

(10) sentimentality, undirected exuberance, and an
almost reverential attitude toward the ideal of
personal freedom for those who already had it.

During that time the culture and civilization of
the American Indian was destroyed, and out of

(15) that time came virtually all the great myths of the
American West—tales of fur traders, mountain
men, steamboat pilots, goldseekers, gamblers,
gunmen, cavalrymen, cowboys, harlots, missionaries,
schoolmarms, and homesteaders. Only

(20) occasionally was the voice of the Indian heard,
and then more often than not it was recorded by
the pen of a white man. The Indian was the dark
menace of the myths, and even if he had known
how to write in English, where would he have

(25) found a printer or a publisher?

Yet they are not all lost, those Indian voices of
the past. A few authentic accounts of American
western history were recorded by Indians either in
pictographs or in translated English, and some

(30) managed to get published in obscure journals,
pamphlets, or books of small circulation. In the
late nineteenth century, when the white man’s
curiosity about Indian survivors of the wars
reached a high point, enterprising newspaper

(35) reporters frequently interviewed warriors and
chiefs and gave them an opportunity to express
their opinions on what was happening in the
West. The quality of these interviews varied
greatly, depending upon the abilities of the interpreters,

(40) or upon the inclination of the Indians to
speak freely. Some feared reprisals for telling the
truth, while others delighted in hoaxing reporters
with tall tales and shaggy-dog stories.
Contemporary newspaper statements by Indians

(45) must therefore be read with skepticism, although
some of them are masterpieces of irony and others
burn with outbursts of poetic fury.

Among the richest sources of first-person
statements by Indians are the records of treaty

(50) councils and other formal meetings with civilian
and military representatives of the United States
government. Isaac Pitman’s new stenographic
system was coming into vogue in the second half
of the nineteenth century, and when Indians spoke

(55) in council a recording clerk sat beside the official
interpreter.

Even when the meetings were in remote parts
of the West, someone usually was available to
write down the speeches, and because of the

(60) slowness of the translation process, much of what
was said could be recorded in longhand.
Interpreters quite often were half-bloods who
knew spoken languages but seldom could read or
write. Like most oral peoples they and the Indians

(65) depended upon imagery to express their thoughts,
so that the English translations were filled with
graphic similes and metaphors of the natural
world. If an eloquent Indian had a poor interpreter,
his words might be transformed to flat

(70) prose, but a good interpreter could make a poor
speaker sound poetic.

Most Indian leaders spoke freely and candidly
in councils with white officials, and as they
became more sophisticated in such matters during

(75) the 1870s and 1880s, they demanded the right to
choose their own interpreters and recorders. In
this latter period, all members of the tribes were
free to speak, and some of the older men chose
such opportunities to recount events they had witnessed

(80) in the past, or sum up the histories of their
peoples. Although the Indians who lived through
this doom period of their civilization have vanished
from the earth, millions of their words are
preserved in official records. Many of the more

(85) important council proceedings were published in
government documents and reports.

Out of all these sources of almost forgotten
oral history, I have tried to fashion a narrative of
the conquest of the American West as the victims

(90) experienced it, using their own words whenever
possible. Americans who have always looked
westward when reading about this period should
read this book facing eastward.

This is not a cheerful book, but history has a

(95) way of intruding upon the present, and perhaps
those who read it will have a clearer understanding
of what the American Indian is, by knowing
what he was. They may learn something about
their own relationship to the earth from a people

(100) who were true conservationists. The Indians knew
that life was equated with the earth and its
resources, that America was a paradise, and they
could not comprehend why the intruders from the
East were determined to destroy all that was

(105) Indian as well as America itself.

13. The author finds the period of 1860–1890 noteworthy because

(A) the journals of the Lewis and Clark expedition were made public during this time

(B) in that period the bulk of original accounts of the “winning of the West” were produced

(C) during these years American Indians made great strides in regaining their lands

(D) only a very few documents dating from this period are still extant

(E) people still believed in personal freedom as an ideal

14. The author most likely uses quotation marks around the word “opening” (line 4) because

(A) the West was closed rather than opened during this period of time

(B) the American West actually was opened for settlement much earlier in the century

(C) from a Native American perspective it is an inaccurate term

(D) he is citing an authoritative source

(E) he has employed the word in its figurative sense

15. A main concern of the author in this passage is to

(A) denounce the white man for his untrustworthiness and savagery

(B) evaluate the effectiveness of the military treaty councils

(C) argue for the improved treatment of Indians today

(D) suggest that Indian narratives of the conquest of the West are similar to white accounts

(E) introduce the background of the original source materials for his text

16. In lines 5–6 “concentration” most nearly means

(A) memory

(B) attention

(C) diligence

(D) imprisonment

(E) accumulation

17. In describing the ideal of freedom revered by the pioneers as “personal freedom for those who already had it” (line 12), the author is being

(A) enthusiastic

(B) ironic

(C) prosaic

(D) redundant

(E) lyrical

18. According to the passage, nineteenth-century newspaper accounts of interviews with Indians may contain inaccuracies for which of the following reasons?

  I. Lack of skill on the part of the translators

 II. The tendency of the reporters to overstate what they were told by the Indians

III. The Indians’ misgivings about possible retaliations

(A) I only

(B) III only

(C) I and II only

(D) I and III only

(E) I, II, and III

19. The author’s tone in describing the Indian survivors can best be described as

(A) skeptical

(B) detached

(C) elegiac

(D) obsequious

(E) impatient

20. The author is most impressed by which aspect of the English translations of Indian speeches?

(A) Their vividness of imagery

(B) Their lack of frankness

(C) The inefficiency of the process

(D) Their absence of sophistication

(E) Their brevity of expression

21. In line 69 “flat” most nearly means

(A) smooth

(B) level

(C) pedestrian

(D) horizontal

(E) unequivocal

22. In treaty councils before 1870, most Indians did not ask for their own interpreters and recorders because

(A) they could not afford to hire people to take down their words

(B) the white officials provided these services as a matter of course

(C) they were unaware that they had the option to demand such services

(D) they preferred speaking for themselves without the help of translators

(E) they were reluctant to have their words recorded for posterity

23. The author most likely suggests that Americans should read this book facing eastward (lines 91–93)

(A) in an inappropriate attempt at levity

(B) out of respect for Western superstitions

(C) in order to read by natural light

(D) because the Indians came from the East

(E) to identify with the Indians’ viewpoint

24. In line 101, “equated with” most nearly means

(A) reduced to an average with

(B) necessarily tied to

(C) numerically equal to

(D) fulfilled by

(E) differentiated by

S T O P

YOU MAY GO BACK AND REVIEW THIS SECTION IN THE REMAINING TIME, BUT DO NOT WORK IN ANY OTHER SECTION UNTIL TOLD TO DO SO.

Section 3

Time—25 Minutes 20 Questions

For each problem in this section determine which of the five choices is correct and blacken the corresponding choice on your answer sheet. You may use any blank space on the page for your work.

Notes:

• You may use a calculator whenever you think it will be helpful.

• Only real numbers are used. No question or answer on this test involves a complex or imaginary number.

• Use the diagrams provided to help you solve the problems. Unless you see the words “Note: Figure not drawn to scale” under a diagram, it has been drawn as accurately as possible. Unless it is stated that a figure is three-dimensional, you may assume it lies in a plane.

• For any function, f, the domain, unless specifically restricted, is the set of all real numbers for which f(x) is also a real number.

Reference Information

123

123

  1. Every Sunday Greg jogs 3 miles. For the rest of the week, each day he jogs 1 mile more than the preceding day. How many miles does Greg jog in 2 weeks?

(A) 42

(B) 63

(C) 84

(D) 98

(E) 117

img

  2. In the figure above, what is the value of x?

(A) 50

(B) 60

(C) 70

(D) 110

(E) It cannot be determined from the information given.

  3. The following table lists the prices of eight types of sandwiches:

img

If the price of a tuna fish sandwich is increased 75¢ and the price of every other sandwich is increased 50¢, how many sandwiches will be more expensive than the tuna fish?

(A) 0

(B) 1

(C) 2

(D) 3

(E) 4

  4. When a gymnast competes at the Olympics, each of six judges awards a score between 0 and 10. The highest and lowest scores are discarded, and the gymnast’s final mark is the average (arithmetic mean) of the remaining scores. What would be a gymnast’s mark if the judges’ scores were 9.6, 9.4, 9.5, 9.7, 9.2, and 9.6?

(A) 9.5

(B) 9.525

(C) 9.55

(D) 9.575

(E) 9.6

img

  5. In parallelogram ABCD above, what is the value of x?

(A)   2

(B)   4

(C)   6

(D) 20

(E) 60

  6. Three lines are drawn in a plane. Which of the following CANNOT be the total number of points of intersection?

(A) 0

(B) 1

(C) 2

(D) 3

(E) They all could.

  7. If a – b = 10, and a2 – b2 = 20, what is the value of b?

(A) –6

(B) –4

(C)   4

(D)   6

(E) It cannot be determined from the information given.

  8. A dealer in rare metals owns 1000 ounces of silver. If every year she sells half of the silver she owns and doesn’t acquire any more, which of the following is an expression for the number of ounces of silver she will own t years from now where t is a positive integer?

(A) img

(B) 1000 × 2t

(C) 1000 × 2t

(D) img

(E) 1000 × 2t

  9. If x = 9 is a solution of the equation x2 – a = 0, which of the following is a solution of x4 – a = 0?

(A) –81

(B)   –3

(C)     0

(D)     9

(E)     81

10. The following table shows the hourly wages earned by the 16 employees of a small company and the number of employees who earn each wage.

Wages per Hour

Number of Employees

$6

3

8

5

10

4

13

4

What is the average (arithmetic mean) of the median and the mode of this set of data?

(A) 4.5

(B)    8

(C) 8.5

(D)    9

(E) 9.5

11. The degree measure of each of the three angles of a triangle is an integer. Which of the following CANNOT be the ratio of their measures?

(A) 2:3:4

(B) 3:4:5

(C) 4:5:6

(D) 5:6:7

(E) 6:7:8

12. If 3x + 2y = 11 and 2x + 3y = 17, what is the average (arithmetic mean) of x and y?

(A) 2.5

(B) 2.8

(C) 5.6

(D) 5.8

(E) 14

Questions 13 and 14 refer to the following definition.

img is a number square if W + Z = X + Y and 2W = 3X.

13. If img is a number square, what is the value of Y?

(A) 0

(B) 2

(C) 4

(D) 6

(E) 8

14. If img is a number square, Y=

(A) img

(B) W

(C) img

(D) 3W

(E) 4W

15. When the price of gold went up, a jeweler raised the prices on certain rings by 60%. On one ring, however, the price was accidentally reduced by 60%. By what percent must the incorrect price be increased to reflect the proper new price?

(A)   60%

(B) 120%

(C) 300%

(D) 400%

(E) It depends on the original price of the ring

img

16. John rode his bicycle 5 miles along a straight road from A to B and back. The graph above shows how far he was from A at any given time. Not counting the time he stopped, what was John’s average speed, in miles per hour, for the round trip?

(A) img

(B) img

(C) img

(D) 10

(E) It cannot be determined from the graph.

17. Let AB, and C be three points in a plane such that AB:BC = 3:5. Which of the following can be the ratio AB:AC?

  I. 1:2

 II. 1:3

III. 3:8

(A) I only

(B) II only

(C) III only

(D) I and III only

(E) I, II, and III

img

18. The semicircle above is the graph of the function

f(x) = img If g(x) is defined by

g(x) = f(3x) + 3, what is g(2)?

(A) img + 3

(B) img + 3

(C) 5

(D) 9

(E) 11

19. A right circular cylinder has a radius of 8 and a height of π2. If a cube has the same volume as the cylinder, what is the length of an edge of the cube?

(A) 4 img

(B) 8 img

(C) 4img img

(D) 4img

(E) 8img

20. On the critical reading portion of the SAT, the raw score is calculated as follows: 1 point is awarded for each correct answer, and img point is deducted for each wrong answer. If Ellen answered all q questions on the test and earned a raw score of 10, how many questions did she answer correctly?

(A) q – 10

(B) img

(C) img − 10

(D) img

(E) 8 + img

S T O P

YOU MAY GO BACK AND REVIEW THIS SECTION IN THE REMAINING TIME, BUT DO NOT WORK IN ANY OTHER SECTION UNTIL TOLD TO DO SO.

Section 4

Time—25 Minutes 35 Questions

Select the best answer to each of the following questions; then blacken the appropriate space on your answer sheet.

Some or all parts of the following sentences are underlined. The first answer choice, (A), simply repeats the underlined part of the sentence. The other four choices present four alternative ways to phrase the underlined part. Select the answer that produces the most effective sentence, one that is clear and exact, and blacken the appropriate space on your answer sheet. In selecting your choice, be sure that it is standard written English, and that it expresses the meaning of the original sentence.

Example:

The first biography of author Eudora Welty came out in 1998 and she was 89 years old at the time.

(A) and she was 89 years old at the time

(B) at the time when she was 89

(C) upon becoming an 89 year old

(D) when she was 89

(E) at the age of 89 years old

img

  1. Because he spoke out against Hitler’s policies was why Dietrich Bonhoeffer, a Lutheran pastor in Nazi Germany, was arrested and eventually hanged by the Gestapo.

(A) Because he spoke out against Hitler’s policies was why Dietrich Bonhoeffer, a Lutheran pastor in Nazi Germany, was arrested and eventually hanged by the Gestapo.

(B) Dietrich Bonhoeffer, a Lutheran pastor in Nazi Germany, was arrested and eventually hanged by the Gestapo because he spoke out against Hitler’s policies.

(C) Because he spoke out against Hitler’s policies, Dietrich Bonhoeffer, a Lutheran pastor in Nazi Germany, was arrested and eventually hung by the Gestapo.

(D) Dietrich Bonhoeffer, a Lutheran pastor in Nazi Germany, being arrested and eventually hung because he spoke out against Hitler’s policies.

(E) A Lutheran pastor in Nazi Germany, Dietrich Bonhoeffer, spoke out against Hitler’s policies so that he arrested and eventually hung.

  2. The difference between Liebniz and Schopenhauer is that the former is optimistic; the latter, pessimistic.

(A) the former is optimistic; the latter, pessimistic

(B) the former is optimistic, the latter, pessimistic

(C) while the former is optimistic; the latter, pessimistic

(D) the former one is optimistic; the latter one is a pessimistic

(E) the former is optimistic; the latter being pessimistic

  3. Most students like to read these kind of books during their spare time.

(A) these kind of books

(B) these kind of book

(C) this kind of book

(D) this kinds of books

(E) those kind of books

  4. John was imminently qualified for the position because he had studied computer programming and how to operate an IBM machine.

(A) imminently qualified for the position because he had studied computer programming and how to operate an IBM machine

(B) imminently qualified for the position since studying computer programming and the operation of an IBM machine

(C) eminently qualified for the position because he had studied computer programming and how to operate an IBM machine

(D) eminently qualified for the position because he had studied computer programming and the operation of an IBM machine

(E) eminently qualified for the position because he has studied computer programming and how to operate an IBM machine

  5. The idea of inoculating people with smallpox to protect them from later attacks was introduced into Europe by Mary Wortley Montagu, who learned of it in Asia.

(A) Mary Wortley Montagu, who learned of it in Asia

(B) Mary Wortley Montagu, who learned of them in Asia

(C) Mary Wortley Montagu, who learned it of those in Asia

(D) Mary Wortley Montagu, learning of it in Asia

(E) Mary Wortley Montagu, because she learned of it in Asia

  6. In general, the fate of Latin American or East Asian countries will affect America more than it does Britain or France.

(A) will affect America more than it does

(B) will effect America more than it does

(C) will affect America more than they do

(D) will effect America more than they do

(E) will affect America more than they would

  7. While campaigning for President, Paul nearly exhausted his funds and must raise money so that he could pay for last-minute television commercials.

(A) exhausted his funds and must raise money so that he could pay

(B) would exhaust his funds to raise money so that he could pay

(C) exhausted his funds and had to raise money so that he can pay

(D) exhausted his funds and had to raise money so that he could pay

(E) exhausted his funds and must raise money so that he can pay

  8. Athletic coaches stress not only eating nutritious meals but also to get adequate sleep.

(A) not only eating nutritious meals but also to get

(B) to not only eat nutritious meals but also getting

(C) not only to eat nutritious meals but also getting

(D) not only the eating of nutritious meals but also getting

(E) not only eating nutritious meals but also getting

  9. The goal of the remedial program was that it enables the students to master the basic skills they need to succeed in regular coursework.

(A) that it enables

(B) by enabling

(C) to enable

(D) where students are enabled

(E) where it enables

  10. Having revised her dissertation with some care, that her thesis advisor rejected the changes distressed her greatly.

(A) that her thesis advisor rejected the changes distressed her greatly

(B) she found her thesis advisor’s rejection of the changes greatly distressing

(C) her thesis advisor’s rejection of the changes was a great distress

(D) she was greatly distressed about her thesis advisor rejecting the changes

(E) her distress at her thesis advisor’s rejection of the changes was great

11. Running an insurance agency left Charles Ives little time for composition, yet he nevertheless developed a unique musical idiom.

(A) nevertheless developed a unique musical idiom

(B) nevertheless developed a very unique musical idiom

(C) therefore developed a uniquely musical idiom

(D) nevertheless developed his musical idiom uniquely

(E) however developed a very unique and idiomatic music

The sentences in this section may contain errors in grammar, usage, choice of words, or idioms. Either there is just one error in a sentence or the sentence is correct. Some words or phrases are underlined and lettered; everything else in the sentence is correct.

If an underlined word or phrase is incorrect, choose that letter; if the sentence is correct, select No error. Then blacken the appropriate space on your answer sheet.

Example:

The region has a climate img plants img rarely img more than twelve inches img

img

12. I img lately about the monsters— or fantasies img whatever—img frightened img as a child. img

13. Postoperative patients img fail img as img as their doctors recommend take longer to img surgery than more active patients do. img

14. He worked in the lumber camps img the summer not img the money img because he wanted to strengthen his muscles by doing img physical labor. img

15. img book is img to become a best seller because it is well written, img and img entertaining. img

16. img a random poll img National Wildlife, the top three threats to the environment img water pollution, air pollution, and img wastes. img

17. His three children, Ruth, Frank, and Ellis, are very talented youngsters, img promise. img

18. img antidrug legislation, calling for more education, and img Bolivia in raids on cocaine dealers are img that the United States is fighting back img “crack” use. img

19. Cajun cooking, which uses img spices, has always been popular in Louisiana, img it is img now becoming known in other img the country. img

20. It seems strange img when Harvey Firestone img the Firestone Tire and Rubber Company in img tires img a novelty. img

21. The same laser technology that is img on compact discs img also img to computers img additional memory. img

22. The Philippine government img Marcos failed img his countrymen that he had won the presidential election, and Corazon Aquino img

23. img who were involved in last week’s img demonstration? img

24. We img any statement about this controversy, img the source, img gossip until img confirmed. img

25. She is the img one of the applicants img img for the position. img

26. img meet publication schedules, publishers often img to trim everyone’s schedule img unexpected problems. img

27. img comeback stories img Lance Armstrong. img

28. A hotel’s img the loyalty of its guests img by the friendliness and courtesy of the employees img are img the front desk. img

29. img scientists are img the philosophical question of what consciousness is, but others restrict img to trying to understand what is going on at the neurological level img consciousness is present. img

The passage below is the unedited draft of a student’s essay. Parts of the essay need to be rewritten to make the meaning clearer and more precise. Read the essay carefully.

The essay is followed by six questions about changes that might improve all or part of the organization, development, sentence structure, use of language, appropriateness to the audience, or use of standard written English. In each case, choose the answer that most clearly and effectively expresses the student’s intended meaning. Indicate your choice by blackening the corresponding space on the answer sheet.

[1] When you turn on the radio or pop in a tape while the house is quiet or going to work or school in your car, you have several choices of music to listen to. [2] Although, in recent years, CDs have become the medium of choice over records and even tapes. [3] On the radio you have your rap on one station, your classical on another, your New Wave music on another, and then you have your Country. [4] Some young people feel that country is for fat old people, but it isn’t. [5] It is music for all ages, fat or thin.

[6] Country music is “fun” music. [7] It has an unmistakable beat and sound that gets you up and ready to move. [8] You can really get into country, even if it is just the clapping of the hands or the stamping of the feet. [9] You can’t help feeling cheerful watching the country performers, who all seem so happy to be entertaining their close “friends,” although there may be 10,000 of them in the stadium or concert hall. [10] The musicians love it, and audience flips out with delight. [11] The interpersonal factors in evidence cause a sudden psychological bond to develop into a temporary, but nevertheless tightly knit, family unit. [12] For example, you can imagine June Carter Cash as your favorite aunt and Randy Travis as your long lost cousin.

[13] Some people spurn country music. [14] Why, they ask, would anyone want to listen to singers whine about their broken marriages or their favorite pet that was run over by an 18-wheeler? [15] They claim that Willie Nelson, one of today’s country legends, can’t even keep his income taxes straight. [16] Another “dynamic” performer is Dolly Parton, whose most famous feature is definitely not her voice. [17] How talented could she be if her body is more famous than her singing?

[18] Loretta Lynn is the greatest. [19] Anyone’s negative feelings towards country music would change after hearing Loretta’s strong, emotional, and haunting voice. [20] Look, it can’t hurt to give a listen. [21] You never know, you might even like it so much that you will go out, pick up a secondhand guitar and learn to strum a few chords.

30. Which is the best revision of the underlined segment of sentence 1 below?

When you turn on the radio or pop in a tape while the house is quiet or going to work or school in your car, you have several choices of music to listen to.

(A) while the house is quiet or in your car going to work or school

(B) driving to work or school while the house is quiet

(C) while the house is quiet or you are driving to work or school

(D) while driving to work or school in your car, and the house is quiet

(E) while there’s quiet in the house or you go to work or school in your car

31. To improve the coherence of paragraph 1, which of the following sentences should be deleted?

(A) Sentence 1

(B) Sentence 2

(C) Sentence 3

(D) Sentence 4

(E) Sentence 5

32. In the context of the sentences that precede and follow sentence 8, which of the following is the best revision of sentence 8?

(A) Clap your hands and stamp your feet is what to do to easily get into country.

(B) You’re really into country, even if it is just clapping of the hands or stamping of the feet.

(C) You can easily get into country just by clapping your hands or stamping your feet.

(D) One can get into country music rather easily; one must merely clap one’s hands or stamp one’s feet.

(E) Getting into country is easy, just clap your hands and stamp your feet.

33. Given the writing style and tone of the essay, which is the best revision of sentence 11?

(A) The interpersonal relationship that develops suddenly creates a temporary, but nevertheless a closely knit, family unit.

(B) A family-like relationship develops quickly and rapidly.

(C) A close family-type relation is suddenly very much in evidence between the performer and his or her audience.

(D) All of a sudden you feel like a member of a huge, but tight, family.

(E) A sudden bond develops between the entertainer and the audience that might most suitably be described as a “family,” in the best sense of the term.

34. Which of the following best describes the function of paragraph 3 in the essay as a whole?

(A) To present some objective data in support of another viewpoint

(B) To offer a more balanced view of the essay’s subject matter

(C) To ridicule readers who disagree with the writer

(D) To lend further support to the essay’s main idea

(E) To divert the reader’s attention from the main idea of the essay

35. Which of the following revisions of sentence 18 provides the smoothest transition between paragraphs 3 and 4?

(A) Loretta Lynn is one of the great singers of country music.

(B) Loretta Lynn, however, is the greatest country singer yet.

(C) But you can bet they’ve never heard Loretta Lynn sing.

(D) The sounds of Loretta Lynn tells a different story, however.

(E) Loretta Lynn, on the other hand, is superb.

S T O P

YOU MAY GO BACK AND REVIEW THIS SECTION IN THE REMAINING TIME, BUT DO NOT WORK IN ANY OTHER SECTION UNTIL TOLD TO DO SO.

Section 6

Time—25 Minutes 24 Questions

Select the best answer to each of the following questions; then blacken the appropriate space on your answer sheet.

Each of the following sentences contains one or two blanks; each blank indicates that a word or set of words has been left out. Below the sentence are five words or phrases, lettered A through E. Select the word or set of words that best completes the sentence.

Example:

Fame is ----; today’s rising star is all too soon tomorrow’s washed-up has-been.

(A) rewarding

(B) gradual

(C) essential

(D) spontaneous

(E) transitory

img

  1. Despite the ---- of the materials with which Tiffany worked, many of his glass masterpieces have survived for more than seventy years.

(A) beauty

(B) translucence

(C) abundance

(D) majesty

(E) fragility

  2. No summary of the behavior of animals toward reflected images is given, but not much else that is ---- seems missing from this comprehensive yet compact study of mirrors and mankind.

(A) redundant

(B) contemplative

(C) relevant

(D) peripheral

(E) disputable

  3. Pain is the body’s early warning system: loss of ---- in the extremities leaves a person ---- injuring himself unwittingly.

(A) agony..incapable of

(B) sensation..vulnerable to

(C) consciousness..desirous of

(D) feeling..habituated to

(E) movement..prone to

  4. Much of the clown’s success may be attributed to the contrast between the ---- manner he adopts and the general ---- that characterizes the circus.

(A) giddy..sobriety

(B) lugubrious..hilarity

(C) gaudy..clamor

(D) joyful..hysteria

(E) frenetic..excitement

  5. Fortunately, she was ---- her accomplishments, properly unwilling to ---- them before her friends.

(A) excited by..parade

(B) immodest about..discuss

(C) deprecatory about..flaunt

(D) uncertain of..concede

(E) unaware of..conceal

Read the passages below, and then answer the questions that follow them. The correct response may be stated outright or merely suggested in the passages.

Questions 6–9 are based on the following passages.

Passage 1

Pioneering conservationist Marjory Stoneman
Douglas called it the River of Grass. Stretching
south from Lake Okeechobee, fed by the raindrenched
Kissimmee River basin, the Everglades

Line (5) is a water marsh, a slow-moving river of swamps
and sawgrass flowing southward to the Gulf of
Mexico. It is a unique ecosystem, whose enduring
value has come from its being home to countless
species of plants and animals: cypress trees and

(10) mangroves, wood storks and egrets, snapping turtles
and crocodiles. For the past 50 years, however,
this river has been shrinking. Never a torrent,
it has dwindled as engineering projects have
diverted the waters feeding it to meet agricultural

(15) and housing needs.

Passage 2

Today South Florida’s sugar industry is in serious
trouble. Responding to the concerns of the
scientific community and to the mandates of the
Everglades Forever Act, local sugar producers

(20) have spent millions of dollars since 1994 to minimize
the runoff of phosphorus from sugar cane
fields into the Everglades. (Phosphorus runoff,
scientists maintain, has encouraged an invasion of
cattails, which overrun the native sawgrass and

(25) choke the flow of water through what was once a
vast sawgrass marsh.) Sugar producers have
adopted ecologically sound farming practices and
at great cost have dramatically reduced phosphorus
levels to help save the Everglades’ fragile

(30) ecosystem. But who or what will help save
Florida’s imperiled sugar industry?

  6. The author of Passage 1 cites the conservationist Marjory Stoneman Douglas in order to

(A) present a viewpoint

(B) challenge an opinion

(C) introduce a metaphor

(D) correct a misapprehension

(E) honor a pioneer

  7. In line 7, “enduring” most nearly means

(A) tolerating

(B) noteworthy

(C) hard-won

(D) lasting

(E) serene

  8. In lines 22–26, the author of Passage 2 uses a parenthetic remark to

(A) cast doubt on the credibility of a statement

(B) provide background on the reasons for a concern

(C) demonstrate support for the scientific community

(D) explain the usage of a technical term

(E) justify the efforts of the sugar industry

  9. On the basis of the final sentence (“But…industry”) of Passage 2, the author of this passage would most likely appear to the author of Passage 1 as

(A) strongly opposed to the Everglades cleanup

(B) well informed concerning specific requirements of the Everglades Forever Act

(C) inclined to overestimate the importance of the sugar industry

(D) having a deep sympathy for environmental causes

(E) having little understanding of scientific methods

Questions 10–15 are based on the following passage.

In this excerpt from Richard Wright’s 1937 novel Black Boy, the young African-American narrator confronts a new world in the books he illegally borrows from the “whites-only” public library.

That night in my rented room, while letting the
hot water run over my can of pork and beans in
the sink, I opened Mencken’s A Book of
Prejudices and began to read. I was jarred and

Line (5) shocked by the style, the clear, clean, sweeping
sentences. Why did he write like that? And how
did one write like that? I pictured the man as a
raging demon, slashing with his pen, consumed
with hate, denouncing everything American,

(10) extolling everything European, laughing at the
weaknesses of people, mocking God, authority.
What was this? I stood up, trying to realize what
reality lay behind the meaning of the words. Yes,
this man was fighting, fighting with words. He

(15) was using words as a weapon, using them as one
would use a club. Could words be weapons?
Well, yes, for here they were. Then, maybe, perhaps,
a Negro could use them as a weapon? No. It
frightened me. I read on, and what amazed me

(20) was not what he said, but how on earth anybody
had the courage to say it.

What strange world was this? I concluded the
book with the conviction that I had somehow
overlooked something terribly important in life. I

(25) had once tried to write, had once reveled in feeling,
had let my crude imagination roam, but the
impulse to dream had been slowly beaten out of
me by experience. Now it surged up again and I
hungered for books, new ways of looking and

(30) seeing. It was not a matter of believing or disbelieving
what I read, but of feeling something new,
of being affected by something that made the look
of the world different.

As dawn broke I ate my pork and beans, feeling

(35) dopey, sleepy. I went to work, but the mood
of the book would not die; it lingered, coloring
everything I saw, heard, did. I now felt that I
knew what the white men were feeling. Merely
because I had read a book that had spoken of how

(40) they lived and thought, I identified myself with
that book. I felt vaguely guilty. Would I, filled
with bookish notions, act in a manner that would
make the whites dislike me?

I forged more notes and my trips to the library

(45) became frequent. Reading grew into a passion.
My first serious novel was Sinclair Lewis’s Main
Street. It made me see my boss, Mr. Gerald, and
identify him as an American type. I would smile
when I saw him lugging his golf bags into the

(50) office. I had always felt a vast distance separating
me from the boss, and now I felt closer to him,
though still distant. I felt now that I knew him,
that I could feel the very limits of his narrow life.
This had happened because I had read a novel

(55) about a mythical man called George F. Babbitt.
But I could not conquer my sense of guilt, my
feeling that the white men around me knew that I
was changing, that I had begun to regard them
differently.

10. The narrator’s initial reaction to Mencken’s prose can best be described as one of

(A) wrath

(B) disbelief

(C) remorse

(D) laughter

(E) disdain

11. To the narrator, Mencken appeared to be all of the following EXCEPT

(A) intrepid

(B) articulate

(C) satiric

(D) reverent

(E) opinionated

12. In line 36, “coloring” most nearly means

(A) reddening

(B) sketching

(C) blushing

(D) affecting

(E) lying

13. The narrator’s attitude in lines 28–30 is best described as one of

(A) dreamy indifference

(B) sullen resentment

(C) impatient ardor

(D) wistful anxiety

(E) quiet resolve

14. The passage suggests that, when he saw Mr. Gerald carrying the golf clubs, the narrator smiled out of a sense of

(A) relief

(B) duty

(C) recognition

(D) disbelief

(E) levity

15. The passage as a whole is best characterized as

(A) an impassioned argument in favor of increased literacy for blacks

(B) a description of a youth’s gradual introduction to racial prejudice

(C) a comparison of the respective merits of Mencken’s and Lewis’s literary styles

(D) an analysis of the impact of ordinary life on art

(E) a portrait of a youth’s response to expanding intellectual horizons

Questions 16–24 are based on the following passage.

The following passage about pond-dwellers is excerpted from a classic essay on natural history written by the zoologist Konrad Lorenz.

There are some terrible robbers in the pond
world, and, in our aquarium, we may witness all
the cruelties of an embittered struggle for existence
enacted before our very eyes. If you have

Line (5) introduced to your aquarium a mixed catch, you
will soon see an example of such conflicts, for,
amongst the new arrivals, there will probably be a
larva of the water-beetle Dytiscus. Considering
their relative size, the voracity and cunning with

(10) which these animals destroy their prey eclipse the
methods of even such notorious robbers as tigers,
lions, wolves, or killer whales. These are all as
lambs compared with the Dytiscus larva.

It is a slim, streamlined insect, rather more

(15) than two inches long. Its six legs are equipped
with stout fringes of bristles, which form broad
oar-like blades that propel the animal quickly and
surely through the water. The wide, flat head
bears an enormous, pincer-shaped pair of jaws

(20) that are hollow and serve not only as syringes for
injecting poison, but also as orifices of ingestion.
The animal lies in ambush on some waterplant;
suddenly it shoots at lightning speed towards its
prey, darts underneath it, then quickly jerks up its

(25) head and grabs the victim in its jaws. “Prey,” for
these creatures, is all that moves or that smells of
“animal” in any way. It has often happened to me
that, while standing quietly in the water of a
pond, I have been “eaten” by a Dytiscus larva.

(30) Even for man, an injection of the poisonous
digestive juice of this insect is extremely painful.

These beetle larvae are among the few animals
that digest “out of doors.” The glandular secretion
that they inject, through their hollow forceps, into

(35) their prey, dissolves the entire inside of the latter
into a liquid soup, which is then sucked in
through the same channel by the attacker. Even
large victims, such as fat tadpoles or dragon-fly
larvae, which have been bitten by a Dytiscus

(40) larva, stiffen after a few defensive moments, and
their inside, which, as in most water animals, is
more or less transparent, becomes opaque as
though fixed by formalin. The animal swells up
first, then gradually shrinks to a limp bundle of

(45) skin that hangs from the deadly jaws, and is finally
allowed to drop. In the confines of an aquarium,
a few large Dytiscus larvae will, within days,
eat all living things over a quarter of an inch long.
What happens then? They will eat each other, if

(50) they have not already done so; this depends less
on who is bigger and stronger than upon who succeeds
in seizing the other first. I have often seen
two nearly equal sized Dytiscus larvae each seize
the other simultaneously and both die a quick

(55) death by inner dissolution. Very few animals,
even when threatened with starvation, will attack
an equal sized animal of their own species with
the intention of devouring it. I only know this to
be definitely true of rats and a few related

(60) rodents; that wolves do the same thing, I am much
inclined to doubt, on the strength of some
observations of which I shall speak later. But Dytiscus
larvae devour animals of their own breed and size,
even when other nourishment is at hand, and that

(65) is done, as far as I know, by no other animal.

16. By robbers (line 1), the author refers to

(A) thieves

(B) plagiarists

(C) people who steal fish

(D) creatures that devour their prey

(E) unethical scientific observers

17. As used in line 5, a “mixed catch” most likely is

(A) a device used to shut the aquarium lid temporarily

(B) a disturbed group of water beetle larvae

(C) a partially desirable prospective denizen of the aquarium

(D) a random batch of creatures taken from a pond

(E) a theoretical drawback that may have positive results

18. The presence of Dytiscus larvae in an aquarium most likely would be of particular interest to naturalists studying

(A) means of exterminating water-beetle larvae

(B) predatory patterns within a closed environment

(C) genetic characteristics of a mixed catch

(D) the effect of captivity on aquatic life

(E) the social behavior of dragon-fly larvae

19. The author’s primary purpose in lines 14–21 is to

(A) depict the typical victim of a Dytiscus larva

(B) point out the threat to humans represented by Dytiscus larvae

(C) describe the physical appearance of an aquatic predator

(D) refute the notion of the aquarium as a peaceful habitat

(E) clarify the method the Dytiscus larva uses to dispatch its prey

20. The passage mentions all of the following facts about Dytiscus larvae EXCEPT that they

(A) secrete digestive juices

(B) attack their fellow larvae

(C) are attracted to motion

(D) provide food for amphibians

(E) have ravenous appetites

21. By digesting “out of doors” (line 33), the author is referring to the Dytiscus larva’s

(A) preference for open-water ponds over confined spaces

(B) metabolic elimination of waste matter

(C) amphibious method of locomotion

(D) extreme voraciousness of appetite

(E) external conversion of food into absorbable form

22. According to the author, which of the following is (are) true of the victim of a Dytiscus larva?

  I. Its interior increases in opacity.

 II. It shrivels as it is drained of nourishment.

III. It is beheaded by the larva’s jaws.

(A) I only

(B) II only

(C) III only

(D) I and II only

(E) II and III only

23. In the final paragraph, the author mentions rats and related rodents in order to emphasize which point about Dytiscus larvae?

(A) Unless starvation drives them, they will not resort to eating members of their own species.

(B) They are reluctant to attack equal-sized members of their own breed.

(C) They are capable of resisting attacks from much larger animals.

(D) They are one of extremely few species given to devouring members of their own breed.

(E) Although they are noted predators, Dytiscus larvae are less savage than rats.

24. The author indicates that in subsequent passages he will discuss

(A) the likelihood of cannibalism among wolves

(B) the metamorphosis of dragon-fly larvae into dragon-flies

(C) antidotes to cases of Dytiscus poisoning

(D) the digestive processes of killer whales

(E) the elimination of Dytiscus larvae from aquariums

S T O P

YOU MAY GO BACK AND REVIEW THIS SECTION IN THE REMAINING TIME, BUT DO NOT WORK IN ANY OTHER SECTION UNTIL TOLD TO DO SO.

Section 7

Time—25 Minutes 18 Questions

You have 25 minutes to answer the 8 multiple-choice questions and 10 student-produced response questions in this section.

For each multiple-choice question, determine which of the five choices is correct and blacken the corresponding choice on your answer sheet. You may use any blank space on the page for your work.

Notes:

• You may use a calculator whenever you think it will be helpful.

• Only real numbers are used. No question or answer on this test involves a complex or imaginary number.

• Use the diagrams provided to help you solve the problems. Unless you see the words “Note: Figure not drawn to scale” under a diagram, it has been drawn as accurately as possible. Unless it is stated that a figure is three-dimensional, you may assume it lies in a plane.

• For any function, f, the domain, unless specifically restricted, is the set of all real numbers for which f(x) is also a real number.

Reference Information

123

123

  1. How many integers are solutions of the inequality 3|x| + 2 < 17?

(A) 0

(B) 4

(C) 8

(D) 9

(E) Infinitely many

  2. If a speed of 1 meter per second is equal to a speed of k kilometers per hour, what is the value of k?

(1 kilometer = 1000 meters)

(A) 0.036

(B) 0.06

(C) 0.36

(D) 0.6

(E) 3.6

3. If f(x)=x2 + img, what is the value of f(–8)?

(A) –66

(B) –62

(C)   62

(D)   64

(E)   66

  4. In 2000, twice as many boys as girls at Adams High School earned varsity letters. From 2000 to 2010, the number of girls earning varsity letters increased by 25% while the number of boys earning varsity letters decreased by 25%. What was the ratio in 2010 of the number of girls to the number of boys who earned varsity letters?

(A) 5/3

(B) 6/5

(C) 1/1

(D) 5/6

(E) 3/5

  5. If today is Saturday, what day will it be 500 days from today?

(A) Saturday

(B) Sunday

(C) Tuesday

(D) Wednesday

(E) Friday

img

  6. If a point is chosen at random from the interior of rectangle ABCD above, what is the probability the point will be in the shaded quadrilateral BDEF?

(A) 1/4

(B) 1/3

(C) 5/12

(D) 1/2

(E) 7/12

  7. If the average (arithmetic mean) of abc, and d is equal to the average of ab, and c, what is d in terms of ab, and c?

(A) a + b + c

(B) img

(C) img

(D) img

(E) img

  8. Because her test turned out to be more difficult than she intended it to be, a teacher decided to adjust the grades by deducting only half the number of points a student missed. For example, if a student missed 10 points, she received a 95 instead of a 90. Before the grades were adjusted, Meri’s grade on the test was A. What was her grade after the adjustment?

(A) img

(B) img

(C) img

(D) img

(E) A + 25

Directions for Student-Produced Response Questions (Grid-ins)

In questions 9–18, first solve the problem, and then enter your answer on the grid provided on the answer sheet. The instructions for entering your answers are as follows:

• First, write your answer in the boxes at the top of the grid.

• Second, grid your answer in the columns below the boxes.

• Use the fraction bar in the first row or the decimal point in the second row to enter fractions and decimal answers.

img

• Grid only one space in each column.

• Entering the answer in the boxes is recommended as an aid in gridding, but is not required.

• The machine scoring your exam can read only what you grid, so you must grid in your answers correctly to get credit.

• If a question has more than one correct answer, grid in only one of these answers.

• The grid does not have a minus sign, so no answer can be negative.

• A mixed number must be converted to an improper fraction or a decimal before it is gridded. Enter 1img as 5/4 or 1.25; the machine will interpret 1 1/4 as 7_4436_SAT_691_748_00148 and mark it wrong.

• All decimals must be entered as accurately as possible. Here are the three acceptable ways of gridding

img

• Note that rounding to .273 is acceptable, because you are using the full grid, but you would receive no credit for .3 or .27, because these answers are less accurate.

  9. Pencils that were selling at three for 25 cents are now on sale at five for 29 cents. How much money, in cents, would you save by buying 60 pencils at the sale price?

10. If 1 < 3x – 5 < 2, what is one possible value for x?

11. What is the largest integer, x, such that x < 10,000 and img is an even integer?

12. Ellie is dropping marbles into a box one at a time in the following order: red, white, white, blue, blue, blue; red, white, white, blue, blue, blue; img How many marbles will be in the box right after the 100th blue one is put in?

13. Four 3-4-5 right triangles and a square whose sides are 5 are arranged to form a second square. What is the perimeter of that square?

Questions 14 and 15 refer to the following definition.

For any positive integer a: «a» = img

14. What is the value of «3» – «4»?

15. What is the ratio of «a + 3» to «a»?

16. Each of 100 cards has none, one, or two of the letters A and C written on it. If 75 cards have the letter A, 30 have the letter C, and fewer than 15 are blank, what is the largest possible number of cards that have both A and Cwritten on them?

17. To use a certain cash machine, you need a Personal Identification Code (PIC). If each PIC consists of two letters followed by one of the digits from 1 to 9 (such as AQ7 or BB3) or one letter followed by two digits (such as Q37 or J88), how many different PIC’s can be assigned?

img

18. In the figure above, the three circles are tangent to one another. If the ratio of the diameter of the large white circle to the diameter of the small white circle is 3:1, what fraction of the largest circle has been shaded?

S T O P

YOU MAY GO BACK AND REVIEW THIS SECTION IN THE REMAINING TIME, BUT DO NOT WORK IN ANY OTHER SECTION UNTIL TOLD TO DO SO.

Section 8

Time—20 Minutes 19 Questions

Select the best answer to each of the following questions; then blacken the appropriate space on your answer sheet.

Each of the following sentences contains one or two blanks; each blank indicates that a word or set of words has been left out. Below the sentence are five words or phrases, lettered A through E. Select the word or set of words that best completes the sentence.

Example:

Fame is ----; today’s rising star is all too soon tomorrow’s washed-up has-been.

(A) rewarding

(B) gradual

(C) essential

(D) spontaneous

(E) transitory

img

  1. Although similar to mice in many physical characteristics, voles may be ---- mice by the shortness of their tails.

(A) distinguished from

(B) classified with

(C) related to

(D) categorized as

(E) enumerated with

  2. Dr. Charles Drew’s technique for preserving and storing blood plasma for emergency use proved so ---- that it became the ---- for the present blood bank system used by the American Red Cross.

(A) irrelevant..inspiration

(B) urgent..pattern

(C) effective..model

(D) innocuous..excuse

(E) complex..blueprint

  3. The likenesses of language around the Mediterranean were sufficiently marked to ---- ease of movement both of men and ideas: it took relatively few alterations to make a Spanish song intelligible in Italy, and an Italian trader could, without much difficulty, make himself at home in France.

(A) eliminate

(B) facilitate

(C) hinder

(D) clarify

(E) aggravate

  4. Because he saw no ---- to the task assigned him, he worked at it in a very ---- way.

(A) function..systematic

(B) method..dutiful

(C) purpose..diligent

(D) end..rigid

(E) point..perfunctory

  5. During the Battle of Trafalgar, Admiral Nelson remained ----, in full command of the situation in spite of the hysteria and panic all around him.

(A) impassable

(B) imperturbable

(C) overbearing

(D) frenetic

(E) lackadaisical

  6. Although he had spent many hours at the computer trying to solve the problem, he was the first to admit that the final solution was ---- and not the ---- of his labor.

(A) trivial..cause

(B) incomplete..intent

(C) adequate..concern

(D) schematic..fault

(E) fortuitous..result

The questions that follow the next two passages relate to the content of both, and to their relationship. The correct response may be stated outright in the passage or merely suggested.

Questions 7–19 are based on the following passages.

The following passages are excerpted from two recent essays that make an analogy between writing and sports. The author of Passage 1, whose manuscript has been rejected by his publisher, discusses the sorts of failures experienced by writers and ballplayers. The author of Passage 2 explores how his involvement in sports affected his writing career.

Passage 1

In consigning this manuscript to a desk drawer,
I am comforted by the behavior of baseball players.
There are no pitchers who do not give up home
runs, there are no batters who do not strike out.

Line (5) There are no major league pitchers or batters who
have not somehow learned to survive giving up
home runs and striking out. That much is obvious.

What seems to me less obvious is how these
“failures” must be digested, or put to use, in the

(10) overall experience of the player. A jogger once
explained to me that the nerves of the ankle are so
sensitive and complex that each time a runner sets
his foot down, hundreds of messages are conveyed
to the runner’s brain about the nature of the

(15) terrain and the requirements for weight distribution,
balance, and muscle-strength. I’m certain
that the ninth-inning home run that Dave
Henderson hit off Donny Moore registered complexly
and permanently in Moore’s mind and

(20) body and that the next time Moore faced
Henderson, his pitching was informed by his
awful experience of October 1986. Moore’s continuing
baseball career depended to some extent
on his converting that encounter with Henderson

(25) into something useful for his pitching. I can also
imagine such an experience destroying an athlete,
registering in his mind and body in such a negative
way as to produce a debilitating fear.

Of the many ways in which athletes and artists

(30) are similar, one is that, unlike accountants or
plumbers or insurance salesmen, to succeed at all
they must perform at an extraordinary level of
excellence. Another is that they must be willing to
extend themselves irrationally in order to achieve

(35) that level of performance. A writer doesn’t have
to write all-out all the time, but he or she must be
ready to write all-out any time the story requires
it. Hold back and you produce what just about any
literate citizen can produce, a “pretty good” piece

(40) of work. Like the cautious pitcher, the timid
writer can spend a lifetime in the minor leagues.

And what more than failure—the strike out,
the crucial home run given up, the manuscript
criticized and rejected—is more likely to produce

(45) caution or timidity? An instinctive response to
painful experience is to avoid the behavior that
produced the pain. To function at the level of
excellence required for survival, writers, like athletes,
must go against instinct, must absorb their

(50) failures and become stronger, must endlessly
repeat the behavior that produced the pain.

Passage 2

The athletic advantages of this concentration,
particularly for an athlete who was making up for
the absence of great natural skill, were considerable.

(55) Concentration gave you an edge over many
of your opponents, even your betters, who could
not isolate themselves to that degree. For example,
in football if they were ahead (or behind) by several
touchdowns, if the game itself seemed to have

(60) been settled, they tended to slack off, to ease off a
little, certainly to relax their own concentration. It
was then that your own unwavering concentration
and your own indifference to the larger point of
view paid off. At the very least you could deal out

(65) surprise and discomfort to your opponents.

But it was more than that. Do you see? The
ritual of physical concentration, of acute engagement
in a small space while disregarding all the
clamor and demands of the larger world, was the

(70) best possible lesson in precisely the kind of selfish
intensity needed to create and to finish a poem, a
story, or a novel. This alone mattered while all the
world going on, with and without you, did not.

I was learning first in muscle, blood, and bone,

(75) not from literature and not from teachers of literature
or the arts or the natural sciences, but from
coaches, in particular this one coach who paid me
enough attention to influence me to teach some
things to myself. I was learning about art and life

(80) through the abstraction of athletics in much the
same way that a soldier is, to an extent, prepared
for war by endless parade ground drill. His body
must learn to be a soldier before heart, mind, and
spirit can.

(85)      Ironically, I tend to dismiss most comparisons
of athletics to art and to “the creative process.”
But only because, I think, so much that is claimed
for both is untrue. But I have come to believe—
indeed I have to believe it insofar as I believe in

(90) the validity and efficacy of art—that what comes
to us first and foremost through the body, as a
sensuous affective experience, is taken and transformed
by mind and self into a thing of the spirit.
Which is only to say that what the body learns

(95) and is taught is of enormous significance—at
least until the last light of the body fails.

  7. Why does the author of Passage 1 consign his manuscript to a desk drawer?

(A) To protect it from the inquisitive eyes of his family

(B) To prevent its getting lost or disordered

(C) Because his publisher wishes to take another look at it

(D) Because he chooses to watch a televised baseball game

(E) To set it aside as unmarketable in its current state

  8. Why is the author of Passage 1 “comforted by the behavior of baseball players” (line 2)?

(A) He treasures the timeless rituals of America’s national pastime.

(B) He sees he is not alone in having to confront failure and move on.

(C) He enjoys watching the frustration of the batters who strike out.

(D) He looks at baseball from the viewpoint of a behavioral psychologist.

(E) He welcomes any distraction from the task of revising his novel.

  9. What function in the passage is served by the discussion of the nerves in the ankle in lines 11–16?

(A) It provides a momentary digression from the overall narrative flow.

(B) It emphasizes how strong a mental impact Henderson’s home run must have had on Moore.

(C) It provides scientific confirmation of the neuromuscular abilities of athletes.

(D) It illustrates that the author’s interest in sports is not limited to baseball alone.

(E) It conveys a sense of how confusing it is for the mind to deal with so many simultaneous messages.

10. In line 18, “registered” most nearly means

(A) enrolled formally

(B) expressed without words

(C) corresponded exactly

(D) made an impression

(E) qualified officially

11. The attitude of the author of Passage 1 to accountants, plumbers, and insurance salesmen (lines 30–33) can best be described as

(A) respectful

(B) cautious

(C) superior

(D) cynical

(E) hypocritical

12. In the final two paragraphs of Passage 1, the author appears to

(A) romanticize the writer as someone heroic in his or her accomplishments

(B) deprecate athletes for their inability to react to experience instinctively

(C) minimize the travail that artists and athletes endure to do their work

(D) advocate the importance of literacy to the common citizen

(E) suggest that a cautious approach would reduce the likelihood of future failure

13. The author of Passage 2 prizes

(A) his innate athletic talent

(B) the respect of his peers

(C) his ability to focus

(D) the gift of relaxation

(E) winning at any cost

14. In line 60, “settled” most nearly means

(A) judged

(B) decided

(C) reconciled

(D) pacified

(E) inhabited

15. What does the author mean by “indifference to the larger point of view” (lines 63 and 64)?

(A) Inability to see the greater implications of the activity in which you were involved

(B) Hostility to opponents coming from larger, better trained teams

(C) Reluctance to look beyond your own immediate concerns

(D) Refusing to care how greatly you might be hurt by your opponents

(E) Being more concerned with the task at hand than with whether you win or lose

16. What is the function of the phrase “to an extent” in line 81?

(A) It denies a situation.

(B) It conveys a paradox.

(C) It qualifies a statement.

(D) It represents a metaphor.

(E) It minimizes a liability.

17. The author finds it ironic that he tends to “dismiss most comparisons of athletics to art” (lines 85 and 86) because

(A) athletics is the basis for great art

(B) he finds comparisons generally unhelpful

(C) he is making such a comparison

(D) he typically is less cynical

(E) he rejects the so-called creative process

18. The authors of both passages would agree that

(A) the lot of the professional writer is more trying than that of the professional athlete

(B) athletics has little to do with the actual workings of the creative process

(C) both artists and athletes learn hard lessons in the course of mastering their art

(D) it is important to concentrate on the things that hurt us in life

(E) participating in sports provides a distraction from the isolation of a writer’s life

19. How would the author of Passage 2 respond to the author of Passage 1’s viewpoint that a failure such as giving up a key home run can destroy an athlete?

(A) An athlete learns through his body that failure is enormously significant and affects him both physically and spiritually.

(B) Athletes of great natural skill suffer less from the agonies of failure than less accomplished athletes do.

(C) If an athlete plays without holding back, he will surpass athletes who are more inherently adept.

(D) If the athlete focuses on the job at hand and not on past errors, he will continue to function successfully.

(E) Athletes are highly sensitive performers who need to be sheltered from the clamor and demands of the larger world.

S T O P

YOU MAY GO BACK AND REVIEW THIS SECTION IN THE REMAINING TIME, BUT DO NOT WORK IN ANY OTHER SECTION UNTIL TOLD TO DO SO.

Section 9

Time—20 Minutes 16 Questions

For each problem in this section determine which of the five choices is correct and blacken the corresponding choice on your answer sheet. You may use any blank space on the page for your work.

Notes:

• You may use a calculator whenever you think it will be helpful.

• Only real numbers are used. No question or answer on this test involves a complex or imaginary number.

• Use the diagrams provided to help you solve the problems. Unless you see the words “Note: Figure not drawn to scale” under a diagram, it has been drawn as accurately as possible. Unless it is stated that a figure is three-dimensional, you may assume it lies in a plane.

• For any function, f, the domain, unless specifically restricted, is the set of all real numbers for which f(x) is also a real number.

Reference Information

123

123

img

  1. In the figure above, lines imgm, and n intersect at the point that is the vertex of each angle. What is the value of y?

(A)   50

(B)   70

(C) 100

(D) 120

(E) 140

  2. In a laboratory a solution was being heated. In 90 minutes, the temperature rose from –8° to 7°. What was the average hourly increase in temperature?

(A)    5°

(B) 7.5°

(C) 10°

(D) 15°

(E) 22.5°

  3. For how many integers, n, is it true that n2 – 30 is negative?

(A)   5

(B)   6

(C) 10

(D) 11

(E) Infinitely many

  4. Which of the following is NOT a solution of 2a2 + 3b = 5?

(A) a = 0 and b = img

(B) a = 1 and b = 1

(C) a = 2 and b = –1

(D) a = 3 and b = –4

(E) a = 4 and b = –9

  5. What is the slope of the line that passes through (0, 0) and is perpendicular to the line that passes through (–2, 2) and (3, 3)?

(A) –5

(B) img

(C)    0

(D)    img

(E)    5

  6. If the measures of the angles of a triangle are in the ratio of 1:2:3, what is the ratio of the lengths of the sides?

(A) 1:2:3

(B) img

(C) img

(D) 3:4:5

(E) It cannot be determined from the information given.

7. A googol is the number that is written as 1 followed by 100 zeros. If g represents a googol, how many digits are there in g2?

(A) 102

(B) 103

(C) 199

(D) 201

(E) 202

img

8. The figure above is the graph of y = f(x). Which of the following is the graph of y = –f(x – 3)?

(A) img

(B) img

(C) img

(D) img

(E) img

9. Which of the following expresses the area of a circle in terms of C, its circumference?

(A) img

(B) img

(C) img

(D) img

(E) img

10. What is the value of img

(A)   2

(B)   4

(C)   8

(D) 16

(E) 64

11. If img – 5 = 2 what is the value of img ?

(A) 2

(B) img

(C) 8

(D) img

(E) 64

12. To get to a business meeting, Joanna drove m miles in h hours, and arrived img hour early. At what rate should she have driven to arrive exactly on time?

(A) img

(B) img

(C) img

(D) img

(E) img

img

13. In the figure above, what is the area of quadrilateral ABCD?

(A) 4

(B) 5

(C) 5.5

(D) 6

(E) 7

14. If y is inversely proportional to x and directly proportional to z, and x = 4 and z = 8 when y = 10, what is the value of x + z when y = 20?

(A)   6

(B) 12

(C) 16

(D) 18

(E) 24

15. What is the average (arithmetic mean) of 330, 360, and 390?

(A) 360

(B) 3177

(C) 310 + 320 + 330

(D) 327 + 357 + 387

(E) 329 + 359 + 389

16. If a and b are the lengths of the legs of a right triangle whose hypotenuse is 10 and whose area is 20, what is the value of (a + b)2?

(A) 100

(B) 120

(C) 140

(D) 180

(E) 200

S T O P

YOU MAY GO BACK AND REVIEW THIS SECTION IN THE REMAINING TIME, BUT DO NOT WORK IN ANY OTHER SECTION UNTIL TOLD TO DO SO.

Section 10

Time—10 Minutes 14 Questions

For each of the following questions, select the best answer from the choices provided and fill in the appropriate circle on the answer sheet.

Some or all parts of the following sentences are underlined. The first answer choice, (A), simply repeats the underlined part of the sentence. The other four choices present four alternative ways to phrase the underlined part. Select the answer that produces the most effective sentence, one that is clear and exact, and blacken the appropriate space on your answer sheet. In selecting your choice, be sure that it is standard written English, and that it expresses the meaning of the original sentence.

Example:

The first biography of author Eudora Welty came out in 1998 and she was 89 years old at the time.

(A) and she was 89 years old at the time

(B) at the time when she was 89

(C) upon becoming an 89 year old

(D) when she was 89

(E) at the age of 89 years old

img

1. Unfortunately, soul singer Anita Baker’s voice has not weathered the years as well as other singers have.

(A) has not weathered the years as well as other singers have

(B) had not weathered the years as well as other singers have

(C) has not been weathered by the years as well as the voices of other singers have been

(D) has not weathered the years as well as other singers’ voices have

(E) has not weathered the years as good as other singers’ voices have

  2. The mathematics teacher drew a right triangle on the blackboard, he proceeded to demonstrate that we could determine the length of the longest side of the triangle if we knew the lengths of its two shorter sides.

(A) The mathematics teacher drew a right triangle on the blackboard, he

(B) The right triangle, which was drawn on the blackboard by the mathematics teacher, he

(C) After drawing a right triangle on the blackboard, the mathematics teacher

(D) A right triangle was first drawn on the blackboard by the mathematics teacher, then he

(E) Once a right triangle was drawn on the blackboard by the mathematics teacher, who then

  3. An inside trader is when a corporate officer who has access to “inside” or privileged information about a company’s prospects uses that information in buying or selling company shares.

(A) when a corporate officer who has access to “inside” or privileged information about a company’s prospects uses that information

(B) when a corporate officer has access to “inside” or privileged information about a company’s prospects and uses that information

(C) a corporate officer who has access to “inside” or privileged information about a company’s prospects and uses that information

(D) a corporate officer who has accessed “inside” or privileged information about a company’s prospects for use of that information

(E) that a corporate officer who has access to “inside” or privileged information about a company’s prospects and he uses that information

  4. Gymnastics students perform stretching exercises to develop flexibility and to become a more agile tumbler.

(A) exercises to develop flexibility and to become a more agile tumbler

(B) exercises for the development of flexibility and to become a more agile tumbler

(C) exercises so that they develop flexibility, becoming a more agile tumbler

(D) exercises to develop flexibility and to become more agile tumblers

(E) exercises because they want to develop flexibility in becoming a more agile tumbler

  5. Because the Ming vase is priceless plus being highly fragile, it is kept safe in a sealed display case.

(A) Because the Ming vase is priceless plus being highly fragile,

(B) Being that the Ming vase is priceless and also it is highly fragile,

(C) Although the Ming vase is priceless and highly fragile,

(D) Because the Ming vase is priceless and highly fragile is why

(E) Because the Ming vase is both priceless and highly fragile,

  6. The soft, pulpy flesh of the passion fruit possesses a flavor at once tart and sweet and the flavor has captivated many prominent chefs, among them Alice Waters.

(A) sweet and the flavor has captivated

(B) sweet that has captivated

(C) sweet that have captivated

(D) sweet and the flavors have captivated

(E) sweet and the favor captivates

  7. Shakespeare’s acting company performed in a relatively intimate setting, appearing before smaller audiences than most theaters today.

(A) appearing before smaller audiences than most theaters today

(B) they appeared before smaller audiences than most theaters today

(C) appearing before audiences smaller than most audiences today

(D) having appeared before smaller audiences than most theaters today

(E) and they appeared before audiences smaller than the ones at most theaters today

  8. Observing the interactions of preschoolers in a playground setting, it can be seen that the less adults relate to the children in their charge, the more these children relate to one another.

(A) Observing the interactions of preschoolers in a playground setting, it can be seen

(B) Having observed the interactions of preschoolers in a playground setting, it can be seen

(C) If one observes the interactions of preschoolers in a playground setting, you can see

(D) Observing the interactions of preschoolers in a playground setting, we can see

(E) Observing the interactions of preschoolers in a playground setting can be seen

  9. A significant percentage of persons summoned for jury service requested a postponement, and only a few were deferred.

(A) A significant percentage of persons summoned for jury service requested a postponement, and only a few were deferred.

(B) A significant percentage of persons, having been summoned for jury service, requested a postponement, a few were only deferred.

(C) A significant percentage of persons summoned for jury service requested a postponement, but only a few were deferred.

(D) After a significant percentage of persons were summoned for jury service, they requested a postponement, only a few being deferred.

(E) Only a few were deferred, a significant percentage of persons summoned for jury service having requested a postponement.

10. Far from being mercenary ambulance chasers, trial lawyers perform a public service by forcing corporations to consider the potential financial cost of pollution, unsafe products, and mistreatment of workers.

(A) Far from being mercenary ambulance chasers

(B) Despite them being mercenary ambulance chasers

(C) Far from them being mercenary ambulance chasers

(D) Far from having been mercenary ambulance chasers

(E) Further from being mercenary ambulance chasers

11. Unsafe at Any Speed is Ralph Nader’s detailed portrait of how the auto industry willfully resisted safety innovations and thus contributed to thousands of highway deaths a year.

(A) portrait of how the auto industry willfully resisted safety innovations and thus contributed to

(B) portrait of when the auto industry was willful about resisting safety innovations and thus contributing to

(C) portrait of how the auto industry fully willed themselves to resist safety innovations and thus contributed to

(D) portrait of how the auto industry willfully resisted safety innovations in order to contribute to

(E) portrait showing how the auto industry willfully resisted safety innovations, and they thus contributed to

12. In 1532, Francisco Pizarro and his troops arrived in Cuzco, took hostage the Incan king, Atahualpa, and then they demanded ransom.

(A) Atahualpa, and then they demanded ransom

(B) who was named Atahualpa, and then they demanded ransom

(C) Atahualpa, it was so they could demand ransom

(D) Atahualpa, and then there was a demand for ransom

(E) Atahualpa, and then demanded ransom

13. Although demand for cars, motorcycles, and other consumer goods are booming, the economy is growing only at roughly 4 percent a year, and the unemployment rate is about 10 percent.

(A) Although demand for cars, motorcycles, and other consumer goods are booming

(B) Because demand for cars, motorcycles, and other consumer goods are booming

(C) Although demand for cars, motorcycles, and other consumer goods is booming

(D) Although demand for cars, motorcycles, and other consumer goods have been booming

(E) Although demand of cars, motorcycles, and other consumer goods is booming

14. Samuel Sewall, who was a judge in the Salem witch trials but later repented his role and, in 1700, wrote the first attack on the American slave trade.

(A) Samuel Sewall, who was a judge in the Salem witch trials but later repented his role and, in 1700,

(B) Samuel Sewall was a judge in the Salem witch trials but who later repented his role and, in 1700,

(C) Samuel Sewall, a judge in the Salem witch trials, but later he repented his role and, in 1700,

(D) Samuel Sewall, a judge in the Salem witch trials, later repented his role and in 1700

(E) Samuel Sewall, who was a judge in the Salem witch trials but who later repented his role, and who, in 1700,

S T O P

YOU MAY GO BACK AND REVIEW THIS SECTION IN THE REMAINING TIME, BUT DO NOT WORK IN ANY OTHER SECTION UNTIL TOLD TO DO SO.

Answer Key

Note: The letters in brackets following the Mathematical Reasoning answers refer to the sections of Chapter 9 in which you can find the information you need to answer the questions. For example, 1. C [E] means that the answer to question 1 is C, and that the solution requires information found in Section 9-E: Averages.

Section 2          Critical Reading

  1. A

  2. D

  3. B

  4. D

  5. D

  6. A

  7. C

  8. C

  9. E

10. B

11. A

12. C

13. B

14. C

15. E

16. E

17. B

18. D

19. C

20. A

21. C

22. C

23. E

24. B

Section 3          Mathematical Reasoning

  1. C [A]

  2. C [I, J]

  3. D [A, Q]

  4. B [E]

  5. C [G, K]

  6. E [I]

  7. B [F]

  8. C [P]

  9. B [G]

10. C [E]

11. E [D, J]

12. B [E, G]

13. E [G]

14. C [G]

15. C [C]

16. B [H, Q]

17. D [D, I]

18. E [R]

19. D [M]

20. E [G, H]

Section 4          Writing Skills

  1. B

  2. A

  3. C

  4. D

  5. A

  6. A

  7. D

  8. E

  9. C

10. B

11. A

12. D

13. C

14. B

15. B

16. C

17. B

18. B

19. A

20. D

21. C

22. C

23. E

24. D

25. C

26. D

27. D

28. A

29. A

30. C

31. B

32. C

33. D

34. B

35. C

Section 5

On this test, Section 5 was the experimental section. It could have been an extra critical reading, mathematics, or writing skills section. Remember: on the SAT you take, the experimental section may be any section from 2 to 7.

Section 6          Critical Reading

  1. E

  2. C

  3. B

  4. B

  5. C

  6. C

  7. D

  8. B

  9. C

10. B

11. D

12. D

13. C

14. C

15. E

16. D

17. D

18. B

19. C

20. D

21. E

22. D

23. D

24. A

Section 7          Mathematical Reasoning

Multiple-Choice Questions

  1. D [A]

  2. E [D]

  3. C [R]

  4. D [C, D]

  5. C [P]

  6. C [O]

  7. B [E, G]

  8. A [E]

Grid-in Questions

img

img

img

Section 8          Critical Reading

  1. A

  2. C

  3. B

  4. E

  5. B

  6. E

  7. E

  8. B

  9. B

10. D

11. C

12. A

13. C

14. B

15. E

16. C

17. C

18. C

19. D

Section 9          Mathematical Reasoning

  1. C [I]

  2. C [E]

  3. D [A, O]

  4. D [G]

  5. A [N]

  6. C [D, J]

  7. D [A, P]

  8. E [R]

  9. A [L]

10. B [A]

11. C [G]

12. E [B, H]

13. A [J, K, N]

14. D [D]

15. E [A, E]

16. D [J, F]

Section 10          Writing Skills

  1. D

  2. C

  3. C

  4. D

  5. E

  6. B

  7. C

  8. D

  9. C

10. A

11. A

12. E

13. C

14. D

Self-Evaluation

Now that you have completed the diagnostic test, evaluate your performance. Identify your strengths and weaknesses, and then plan a practical study program based on what you have discovered. Follow these steps to evaluate your work on the diagnostic test. (Note: You’ll find the charts referred to in steps 1–5 on the next few pages.)

img STEP 1 Use the answer key to check your answers for each section.

img STEP 2 For each section, count the number of correct and incorrect answers (remember that you don’t count omitted answers), and enter the numbers on the appropriate lines of the chart “Calculate Your Raw Score.” Then do the indicated calculations to get your Critical Reading Raw Score and your Mathematical Reasoning Raw Score.

img STEP 3 Consult the chart “Evaluate Your Performance” to see how well you did.

img STEP 4 To pinpoint the specific areas in which you need to improve, circle the numbers of the questions that you either left blank or got wrong on the “Identify Your Weaknesses” charts. You can then see where to concentrate your efforts to get the most out of your study time. The chart for the math sections gives you page references for review and practice by skill areas. The charts for the critical reading and writing skills sections refer you to the appropriate chapters to study for each question type.

img STEP 5 Wherever you had a concentration of circles, do the review and practice indicated on the charts.

Important: Remember that, in addition to evaluating your scores, you should read all of the answer explanations for questions you answered incorrectly, questions you omitted, and questions you answered correctly but found difficult. Reviewing the answer explanations will help you understand concepts and strategies, and may point out shortcuts.

Score Your Own SAT Essay

Use this table as you rate your performance on the essay-writing section of this Model Test. Circle the phrase that most accurately describes your work. Enter the numbers in the scoring chart below. Add the numbers together and divide by 6 to determine your total score. The higher your total score, the better you are likely to do on the essay section of the SAT.

Note that on the actual SAT two readers will rate your essay; your essay score will be the sum of their two ratings and could range from 12 (highest) to 2 (lowest). Also, they will grade your essay holistically, rating it on the basis of their overall impression of its effectiveness. They will not analyze it piece by piece, giving separate grades for grammar, vocabulary level, and so on. Therefore, you cannot expect the score you give yourself on this Model Test to predict your eventual score on the SAT with any great degree of accuracy. Use this scoring guide instead to help you assess your writing strengths and weaknesses, so that you can decide which areas to focus on as you prepare for the SAT.

Like most people, you may find it difficult to rate your own writing objectively. Ask a teacher or fellow student to score your essay as well. With his or her help you should gain added insights into writing your 25-minute essay.

img

Self-Scoring Chart

Scoring Chart (Second Reader)

For each of the following categories, rate the essay from 1 (lowest) to 6 (highest)

For each of the following categories, rate the essay from 1 (lowest) to 6 (highest)

Position on the Topic

______

Position on the Topic

______

Organization of Evidence

______

Organization of Evidence

______

Sentence Structure

______

Sentence Structure

______

Level of Vocabulary

______

Level of Vocabulary

______

Grammar and Usage

______

Grammar and Usage

______

Overall Effect

______

Overall Effect

______

TOTAL

______

TOTAL

______

(To get a score, divide the total by 6)

______

(To get a score, divide the total by 6)

______

Calculate Your Raw Score

Critical Reading

img

img

img

Mathematical Reasoning

img

Mathematical Reasoning Raw Score = (D) + (E) + (F) + (G) = ____________

Writing Skills

img

Writing Skills Raw Score = H + I (J is a separate subscore)

Evaluate Your Performance

img

Identify Your Weaknesses

img

Identify Your Weaknesses

img

Identify Your Weaknesses

img

Answers Explained

Section 2 Critical Reading

  1. (A).    Because lightning victims are so battered and confused, they seem like assault victims. Thus, they are often mistaken for victims of assault.

(Cause and Effect Signal)

  2. (D).    Anyone who has produced more than three hundred books in a single lifetime is an enormously productive or prolific writer. Writers are often described as prolific, but few, if any, have been as prolific as the late Dr. Asimov. Beware of Eye-Catchers: Choice A is incorrect. Fastidious means painstakingly careful; it has nothing to do with writing quickly.

(Examples)

  3. (B).    Time limitations would cause problems for you if you were reading a lengthy book. To save time, you might want to read it in an abridged or shortened form. Remember to watch for signal words that link one part of the sentence to another. The use of “because” in the opening clause is a cause signal.

(Cause and Effect Signal)

  4. (D).    Speakers wish to communicate unambiguously in order that there may be no confusion about their meaning. Remember to watch for signal words that link one part of the sentence to another. The presence of “and” linking two items in a pair indicates that the missing word may be a synonym or near-synonym for the other linked word. In this case, unambiguously is a synonym for clearly. Similarly, the use of “so that” in the second clause signals cause and effect.

(Argument Pattern)

  5. (D).    Lisa was normally gregarious or sociable. When she unexpectedly lost her job, she became quiet and withdrawn (distant; unsociable). Note how the signal word Although indicates a contrast between her normally sociable and later unsociable states.

(Contrast Pattern)

  6. (A).    Wilma Mankiller, a female, heads a major American Indian tribe. She performs her role successfully: she is “increasingly popular.” By her success, she has shattered or exploded a myth of male supremacy.

(Argument Pattern)

  7. (C).    The commission censured or condemned the senator for doing something wrong: his expenditures of public funds were lavish or extravagant. He spent the public’s money in an unjustifiable, unwarranted way.

(Cause and Effect Pattern)

  8. (C).    A stereotyped or oversimplified portrait of a slave would lead sensitive readers to express criticism because the issue of slavery was treated so casually. Thus, they normally would be detractors of the novel. However, Huckleberry Finn is such a fine work that even its critics acknowledge its greatness. Signal words are helpful here. “Despite” in the first clause implies a contrast, and “even” in the second clause implies that the subjects somewhat reluctantly agree that the novel is a masterpiece.

(Contrast Signal)

  9. (E).    The final sentence of the passage maintains that, contrary to expectation, the jellyfish has a sophisticated or complex genetic structure. Beware of eye-catchers. Choice B is incorrect. “Spineless” (line 1) here means invertebrate, lacking a backbone or spinal column. It does not mean cowardly.

10. (B).    The second sentence of the passage states that the jellyfish “seems the most primitive of creatures.” The last sentence of the passage, however, contradicts or denies that assumption.

11. (A).    The human faculty for myth is the capacity or ability of people to invent legends.

12. (C).    The fact that Raleigh is remembered more for a romantic, perhaps apocryphal, gesture than for his voyages of exploration illustrates how legendary events outshine historical achievements in the public’s mind.

13. (B).    The author is writing a book about the effect of the opening of the West on the Indians living there. As a historian, he needs primary source materials—firsthand accounts of the period written by men and women living at that time. Thus, he finds the period of 1860–1890 worth mentioning because during those years the “greatest concentration of recorded experience and observation” (the bulk of original accounts) was created.

14. (C).    Only the white settlers looked on their intrusion into Indian territory as the opening of the West. To the Native Americans, it was an invasion. Thus, “opening” from a Native American perspective is an inaccurate term.

15. (E).    Throughout the passage the author presents and comments on the nature of the original documents that form the basis for his historical narrative. Thus, it is clear that a major concern of his is to introduce these “sources of almost forgotten oral history” to his readers. Choice A is incorrect. The author clearly regrets the fate of the Indians. However, he does not take this occasion to denounce or condemn the white man. Choice B is incorrect. While the author discusses the various treaty councils, he does not evaluate or judge their effectiveness. Choice C is incorrect. The author never touches on the current treatment of Indians. Choice D is incorrect. The author indicates no such similarity.

16. (E).    Of all the thousands of published descriptions of the opening of the West, the greatest concentration or accumulation of accounts dates from the period of 1860 to 1890.

17. (B).    The author is describing a period in which Native Americans lost their land and much of their personal freedom to the same pioneers who supposedly revered the ideal of freedom. Thus, in describing the ideal of freedom revered by the pioneers as “personal freedom for those who already had it” (in other words, personal freedom for the pioneers, not the Indians), the author is being ironic.

18. (D).    You can arrive at the correct choice by the process of elimination. Statement I is true. The passage states that the quality of the interviews depended on the interpreters’ abilities. Inaccuracies could creep in because of the translators’ lack of skill. Therefore, you can eliminate Choice B. Statement II is untrue. The passage indicates that the Indians sometimes exaggerated, telling the reporters tall tales. It does not indicate that the reporters in turn overstated what they had been told. Therefore, you can eliminate Choices C and E. Statement III is true. The passage indicates that the Indians sometimes were disinclined to speak the whole truth because they feared reprisals (retaliation) if they did. Therefore, you can eliminate choice A. Only Choice D is left. It is the correct answer.

19. (C).    Brown speaks of the Indians who lived through the “doom period of their civilization,” the victims of the conquest of the American West. In doing so, his tone can best be described as elegiac, expressing sadness about their fate and lamenting their vanished civilization.

20. (A).    In the fifth paragraph Brown comments upon the “graphic similes and metaphors of the natural world” found in the English translations of Indian speeches. Thus, he is impressed by their vividness of imagery.

21. (C).    Commenting about inadequate interpreters who turned eloquent Indian speeches into “flat” prose, Brown is criticizing the translations for their pedestrian, unimaginative quality.

22. (C).    Lines 73–76 state that, as the Indian leaders be came more sophisticated or knowledgeable about addressing treaty councils, “they demanded the right to choose their own interpreters and recorders.” Until they had become familiar with the process, they were unaware that they had the option to demand such services.

23. (E).    Brown has tried to create a narrative of the winning of the West from the victims’ perspective. In asking his readers to read the book facing eastward (the way the Indians would have been looking when they first saw the whites headed west), he is asking them metaphorically to identify with the Indians’ viewpoint.

24. (B).    In the sentence immediately preceding the one in which the phrase “equated with” appears, Brown calls the Indians “true conservationists.” Such conservationists know that life is necessarily tied to the earth and to its resources, and that by destroying these resources, by imbalancing the equation, so to speak, “the intruders from the East” would destroy life itself.

Section 3 Mathematical Reasoning

  1. (C).    Just quickly add up the number of miles Greg jogs each week:

3 + 4 + 5 + 6 + 7 + 8 + 9 = 42.

In 2 weeks he jogs 84 miles.

img

  2. (C).    In the figure above, x + y + z = 180. Also, since y = 60 (180 – 120) and z = 50 (180 – 130), then

x = 180 – (50 + 60) = 180 – 110 = 70.

  3. (D).    After an increase of 75¢, a tuna fish sandwich will cost $5.00. The only sandwiches that, after a 50¢ increase, will be more expensive than the tuna fish are the 3 that now cost more than $4.50.

  4. (B)img    Discard the scores of 9.2 and 9.7, and take the average of the other four scores:

img

  5. (C).    The sum of the measures of two adjacent angles of a parallelogram is 180°. Therefore, 180 = 10x + 25x – 30 = 35x – 30, which implies that 35x = 210 and x = 6.

  6. (E).    The figures below show that all of the choices are possible.

img

  7. (B).    Since a2 – b2 = (a – b)(a + b), then: 20 = a2 – b2 = (a – b)(a + b) = 10(a + b). Therefore, a + b = 2. Adding the equations a + b = 2 and a – b = 10 gives

2a = 12 ⇒ a = 6 ⇒ b = –4.

  8. (C).    Each year the dealer sells half of her silver, so after 1 year she owns img 1000 ounces. After 2 years she owns half as many ounces:

img

In general, after t years, she will own

img ounces.

Since img

img and img

  9. (B).    Since 9 is a solution of x2 – a = 0, then 92 – a = 0 ⇒ 81 – a = 0 ⇒ a = 81. Now solve the equation x4 – a = 0. img

10. (C).    The mode is 8, since more people earn $8 an hour than any other salary. Also, since there are 16 employees, the median is the average of the 8th and 9th items of data: $8 and $10, so the median is 9. Finally, the average of 8 and 9 is 8.5.

11. (E).    If the ratio were a:b:c, then 180 = ax + bx + cx = (a + b + c)x. Since in each of the choices the ratio is written in lowest terms, a + b + c must be a factor of 180. This is the case in choices A through D. Only Choice E, 6:7:8, fails: 6 + 7 + 8 = 21, which is not a divisor of 180.

12. (B).    Add the two equations to get 5x + 5y = 28. Then dividing each term by 5, we get img The average of x and y is img

13. (E).    Since W = 3 and 2W = 3X, then 3X = 6 ⇒ X = 2. Therefore img

14. (C).    By definition, img but the definition also states that 2W = 3X, so img. Therefore img

15. (C).    If the ring was originally priced at $100, it was accidentally marked $40 instead of $160. The incorrect price of $40 must be increased by $120, which is 3 times, or 300% of, the incorrect price.

16. (B).    John’s average speed is calculated by dividing his total distance of 10 miles by the total time he spent riding his bicycle. Each tick mark on the horizontal axis of the graph represents 10 minutes. He left at 8:30 and arrived back home img hours later, at 10:00. However, he stopped for 10 minutes, from 9:20 to 9:30, so he was riding for only 1 hour and 20 minutes, or img hours. Finally, img

17. (D).    Assume AB = 3 and BC = 5. The least that AC can be is 2, if A is on line, img, between B and C; and the most AC can be is 8, if A is on line img, so that B is between A and C. If A is not on line imgAC can be any length between 2 and 8.

img

Therefore the ratio AB:AC can be any number between 3:2 (= 1.5) and 3:8 (= 0.375). In particular, it can be 1:2 (= 0.5) and 3:8. (I and III are true.) It cannot be 1:3 (= 0.333). (II is false.) Statements I and III only are true.

18. (E).    g(2) = f(3 × 2) + 3 = f (6) + 3. img So g(2) = f (6) + 3 = 8 + 3 = 11.

19. (D).    The formula for the volume of a cylinder is V = img r2h. Replacing r by 8 and h by img2, we get that V = img (8)2(img 2) = 64img3. If e is the edge of the cube, then the volume of the cube is e3. So e3 = 64img3 ⇒ e = 4img.

20. (E).    To earn 10 points, Ellen needed to get 10 correct answers and then earn no more points on the remaining q – 10 questions. To earn no points on a set of questions, she had to miss 4 questions (thereby losing imgpoint) for every 1 question she got right in that set. She answered img of the q – 10 questions correctly (and img of them incorrectly). The total number of correct answers was imgAlternative solution: Let c be the number of questions Ellen answered correctly, and q – c the number she missed. Then her raw score is img which equals 10, so img So, img

Section 4 Writing Skills

  1. (B).    Choice B eliminates the excessive wordiness of the original sentence without introducing any errors in diction.

  2. (A).    As used in Choice A, the semicolon separating a pair of clauses is correct. Choices C–E introduce errors in parallel structure.

  3. (C).    Error in agreement. Kind is singular and requires a singular modifier (this).

  4. (D).    Choice D corrects the error in diction (eminently, not imminently) and the error in parallel structure.

  5. (A).    The original answer provides the most effective and concise sentence.

  6. (A).    The original sentence is correct. The singular pronoun it refers to the subject of the main clause, fate (singular).

  7. (D).    Choices A, B, C, and E suffer from errors in the sequence of tenses.

  8. (E).    Error in parallelism. There is a lack of parallel structure in the other four choices.

  9. (C).    Errors in precision and clarity. Choice A states the result of the program rather than the goal. Choice B results in a sentence fragment. Choices D and E use the was where construction, which is unclear and should be avoided.

10. (B).    Dangling modifier. Ask yourself who revised the dissertation. Clearly, she (the writer) did.

11. (A).    Sentence is correct. Unique means being without a like or equal. Avoid phrases like very unique and more unique that imply there can be degrees of uniqueness.

12. (D).    The reflexive pronoun myself cannot be used as the object of the verb frightened. Change myself to me.

13. (C).    Adjective and adverb confusion. Change regular to regularly.

14. (B).    Error in parallelism. Change not because of the money to not because he needed the money (a clause) to parallel the clause that follows but.

15. (B).    Error in diction. Change liable to likely.

16. (C).    Error in subject-verb agreement. Change is to are.

17. (B).    Error in diction. Latter should not be used to refer to more than two items. Change latter to last.

18. (B).    Error in parallelism. Change to aid to aiding.

19. (A).    Adjective and adverb confusion. Change special prepared to specially prepared.

20. (D).    Error in tense. Change had been to were.

21. (C).    Error in parallelism. Change under application to being applied.

22. (C).    Faulty verbal. Change satisfying to the infinitive to satisfy.

23. (E).    Sentence is correct.

24. (D).    Error in pronoun-antecedent agreement. Change they are to it is.

25. (C).    Error in subject-verb agreement. The antecedent of who is one. Therefore, who is is correct.

26. (D).    Error in parallelism. Change and leaving room for to to leave room for.

27. (D).    Incomplete comparison. Compare stories with stories, not stories with champion. The sentence should read: “There are probably few comeback stories as moving as that of cycling’s stalwart champion, Lance Armstrong.”

28. (A).    Unidiomatic preposition. Replace ability for winning with ability to win.

29. (A).    Error in coordination and subordination. Remember: any sentence elements that are not underlined are by definition correct. Here, the coordinating conjunction but is not underlined. Coordinating conjunctions connect sentence elements that are grammatically equal. In this case, but should connect the main clause beginning “others restrict themselves” with another main clause. However, while, a subordinating conjunction, introduces a subordinate clause, not a main clause. To correct the error, delete While and begin the sentence Some scientists are absorbed.

30. (C).    Choice A says that the house is in your car, an unlikely situation. Choice B contains an idea that the writer could not have intended. Choice C accurately states the intended idea. It is the best answer. Choice D, like Choice B, contains an idea that is quite absurd. Choice E is wordy and awkwardly expressed.

31. (B).    All the sentences except sentence 2 contribute to the development of the essay’s topic. Therefore, Choice B is the best answer.

32. (C).    Choice A is awkwardly expressed. Choice B is awkward and contains the pronoun it, which has no specific antecedent. Choice C is accurately expressed and is consistent with the sentences that precede and follow sentence 8. It is the best answer. Choice D is written in a style that is different from that of the rest of the essay. Choice E would be a good choice, but it contains a comma splice. A comma may not be used to join two independent clauses.

33. (D).    Choice A is quite formal and is not in keeping with the style and tone of the essay. Choice B is close to the style and tone of the essay, but it contains the redundancy quickly and rapidly. Choice C has a formal tone inconsistent with the rest of the essay. Choice D uses the second-person pronoun and is consistent with the folksy, conversational style of the essay. It is the best answer. Choice E uses an objective tone far different from the writing in the rest of the essay.

34. (B).    Choice A is only partly true. While the paragraph gives another viewpoint, the data it contains are hardly objective. Choice B accurately states the writer’s intention. It is the best answer. Choices C, D, and E in no way describe the function of paragraph 3.

35. (C).    Choice A provides no particular link to the preceding paragraph. Choice B provides a rather weak transition between paragraphs. Choice C creates a strong bond between paragraphs by alluding to material in paragraph 3 and introducing the topic of paragraph 4. It is the best answer. Choice D could be a good transition were it not for the error in subject-verb agreement. The subject sounds is plural; the verb tells is singular. Choice E provides a weak transition and its writing style is not consistent with the rest of the essay.

Section 6 Critical Reading

  1. (E).    Tiffany’s works of art have survived in spite of their fragility (tendency to break). Remember to watch for signal words that link one part of the sentence to another. The use of “despite” in the opening phrase sets up a contrast. Despite signals you that Tiffany’s glass works were unlikely candidates to survive for several decades.

(Contrast Signal)

  2. (C).    A comprehensive or thorough study would not be missing relevant or important material. Remember to watch for signal words that link one part of the sentence to another. The use of “but” in the second clause sets up a contrast.

(Contrast Signal)

  3. (B).    Pain is a sensation. Losing the ability to feel pain would leave the body vulnerable, defenseless, lacking its usual warnings against impending bodily harm. Note how the second clause serves to clarify or explain what is meant by pain’s being an “early warning system.”

(Definition)

  4. (B).    A lugubrious (exaggeratedly gloomy) manner may create laughter because it is so inappropriate in the hilarity (noisy gaiety) of the circus. The clown’s success stems from a contrast. The missing words must be antonyms or near-antonyms. You can immediately eliminate Choices C, D, and E as nonantonym pairs. In addition, you can eliminate Choice A; sobriety or seriousness is an inappropriate term for describing circus life.

(Contrast Pattern)

  5. (C).    If she was deprecatory about her accomplishments (diminished them or saw nothing praiseworthy in them), she would be unwilling to boast about them or flaunt them. Note the use of “properly” to describe her unwillingness to do something. This suggests that the second missing word would have negative associations.

(Definition)

  6. (C).    The author refers to Douglas in order to introduce Douglas’s metaphoric description of the Everglades as the River of Grass.

  7. (D).    Enduring value is value that lasts. The lasting value of the Everglades is that it provides a habitat for endangered species.

  8. (B).    The author’s parenthetic remark serves to provide background on the reasons for the scientific and governmental concern about the dangers of phosphorus runoff.

  9. (C).    The author of Passage 1 is wholly concerned with the threat to the Everglades’ fragile ecosystem. The environment is what is important to her. She mentions agricultural needs only in terms of how they have affected the River of Grass. Given her perspective, she would most likely view the author of Passage 2 as someone inclined to overestimate the importance of the sugar industry.

10. (B).    The author describes himself as “jarred and shocked” (lines 4 and 5). He asks himself, “What strange world was this?” His initial reaction to Mencken’s prose is one of disbelief. Choice A is incorrect. Mencken rages; the narrator does not. Choice C is incorrect. It is unsupported by the passage. Choices D and E are incorrect. Again, these terms apply to Mencken, not to the narrator.

11. (D).    The narrator does not portray Mencken as reverent or respectful of religious belief. Instead, he says that Mencken mocks God. Choice A is incorrect. The narrator portrays Mencken as intrepid (brave); he wonders where Mencken gets his courage. Choice B is incorrect. The narrator portrays Mencken as articulate (verbally expressive); he says Mencken writes clear, clean sentences. Choice C is incorrect. The narrator portrays Mencken as satiric (mocking); he says Mencken makes fun of people’s weaknesses. Choice E is incorrect. The narrator portrays Mencken as opinionated (stubborn about his opinions; prejudiced). Mencken’s book, after all, is A Book of Prejudices. Remember: when asked about specific information in the passage, spot key words in the question and scan the passage to find them (or their synonyms).

12. (D).    The mood of the book colored or affected the narrator’s perceptions. Remember: when answering a vocabulary-in-context question, test each answer choice, substituting it in the sentence for the word in quotes.

13. (C).    The narrator feels a hunger for books that surges up in him. In other words, he is filled with impatient ardor or eagerness. Choice A is incorrect. The narrator has his dreams, but he is involved rather than indifferent. Choices B and D are incorrect. There is nothing in the lines to suggest them. Choice E is incorrect. The narrator is determined, but his resolve is active and eager rather than quiet. Remember: when asked to determine the author’s attitude or tone, look for words that convey emotion or paint pictures.

14. (C).    The narrator is able to identify Mr. Gerald as an American type. He feels closer to Mr. Gerald, familiar with the limits of his life. This suggests that he smiles out of a sense of recognition. Choices A, B, D, and E are incorrect. There is nothing in the passage to suggest them. Remember: when asked to make inferences, base your answers on what the passage implies, not what it states directly.

15. (E).    Phrases like “of feeling something new, of being affected by something that made the look of the world different” and “filled with bookish notions” reflect the narrator’s response to the new books he reads. You have here a portrait of a youth’s response to his expanding intellectual horizons. Choice A is incorrect. The narrator is not arguing in favor of a cause; he is recounting an episode from his life. Choice B is incorrect. The narrator was aware of racial prejudice long before he read Mencken. Choice C is incorrect. The passage is not about Mencken’s and Lewis’s styles; it is about their effect in opening up the world to the narrator. Choice D is incorrect. The passage is more about the impact of art on life than about the impact of life on art. Remember: when asked to find the main idea, be sure to check the opening and summary sentences of each paragraph.

16. (D).    The terrible robbers in the pond world are the cruel creatures that, in the course of the struggle to exist, devour their fellows.

17. (D).    Here, “catch” is used as in fishing: “a good catch of fish.” Suppose you want to collect a sample of pond-dwellers. You lower a jar into the nearest pond and capture a random batch of creatures swimming by—fish, tadpoles, full-grown insects, larvae—in other words, a “mixed catch.”

18. (B).    The opening paragraph states that the introduction of the Dytiscus larvae to the aquarium will result in a struggle for existence in which the larvae will destroy their prey. The larvae, thus, are predators (hunters of prey). This suggests that their presence would be of particular interest to naturalists studying predatory patterns at work within a closed environment such as an aquarium.

19. (C).    The author is describing how the Dytiscus larva looks: slim body, six legs, flat head, huge jaws. Choice A is incorrect. All the details indicate the author is describing the killer, not the victim.

20. (D).    Though the passage mentions amphibians— tadpoles—and food, it states that the tadpoles provide food for the larvae, not vice versa. The passage nowhere states that the larvae are a source of food for amphibians. Choice A is incorrect. The passage states that the larvae secrete digestive juices; it mentions secretion in line 33. Choice B is incorrect. The passage states that the larvae attack one another; they seize and devour their own breed (lines 53–63). Choice C is incorrect. The passage states that the larvae are attracted to motion; prey for them “is all that moves.” Choice E is incorrect. The passage states that the larvae have ravenous appetites: their “voracity” is unique. Remember: when asked about specific information in the passage, spot key words in the question and scan the passage to find them (or their synonyms).

21. (E).    Digesting “out of doors” refers to the larva’s external conversion of food into absorbable form. Look at the sentence immediately following line 33. Break down the process step by step. The larva injects a secretion into the victim. The secretion dissolves the victim’s “entire inside.” That is the start of the digestive process. It takes place inside the victim’s body; in other words, outside the larva’s body—“out of doors.” Only then does the larva begin to suck up the dissolved juices of his prey.

22. (D).    Choice D is correct. You can arrive at it by the process of elimination. Statement I is true. The inside of the victim “becomes opaque” (line 42); it increases in opacity. Therefore, you can eliminate Choices B, C, and E. Statement II is also true. As the victim is drained, its body shrivels or “shrinks to a limp bundle of skin.” Therefore, you can eliminate choice A. Statement III has to be untrue. The victim’s head must stay on; otherwise, the dissolving interior would leak out. Only Choice D is left. It is the correct answer.

23. (D).    The author mentions rats because a rat will attack and devour other rats. He is sure rodents do this; he’s not sure any other animals do so. Thus, he mentions rats and related rodents to point up an uncommon characteristic also found in Dytiscus larvae.

24. (A).    In lines 61 and 62 the author mentions some “observations of which I shall speak later.” These observations deal with whether wolves try to devour other wolves. Thus, the author clearly intends to discuss the likelihood of cannibalism among wolves. In answering questions about what may be discussed in subsequent sections of the text, pay particular attention to words that are similar in meaning to subsequent: followingsucceedingsuccessivelater.

Section 7 Mathematical Reasoning

MULTIPLE-CHOICE QUESTIONS

  1. (D).    img There are 9 integers whose absolute values are less than 5: –4, –3, –2, –1, 0, 1, 2, 3, 4.

  2. (E).    Set up a proportion:

img

Cross-multiplying the first and last ratios, you get 10k = 36, and so k = 3.6 .

  3. (C)img    img

  4. (D).    Pick easy-to-use numbers. Assume that, in 2000, 200 boys and 100 girls earned varsity letters. Then, in 2010, there were 150 boys and 125 girls. The ratio of girls to boys was img

  5. (C)img    The days of the week form a repeating sequence with 7 terms in the set that repeats. The nth term is the same as the r th term, where r is the remainder when n is divided by 7. [See Section P in Chapter 9.] img the quotient is 71. 71 × 7 = 497 and img the remainder is 3.

Therefore, 500 days from Saturday will be the same day as 3 days from Saturday, namely Tuesday.

  6. (C).    The area of rectangle ABCD = (4)(6) = 24. The area of right triangle img and the area of right triangle img Therefore, the area of quadrilateral BDEF = 24 – 12 – 2 = 10. Then the shaded area is img the area of the rectangle, and so the required probability is img

  7. (B).    The easiest observation is that, if adding a fourth number, d, to a set doesn’t change the average, then d is equal to the existing average. If you don’t realize that, solve for d:

img

  8. (A).    If Meri earned a grade of A, she missed (100 – A) points. In adjusting the grades, the teacher decided to deduct only half that number: img so Meri’s new grade was img

GRID-IN QUESTIONS

  9. (152) img Normally, to get 60 pencils you would need to buy 20 sets of three at 25 cents per set, a total expenditure of 20 × 25 = 500 cents. On sale, you could get 60 pencils by buying 12 sets of five at 29 cents per set, for a total cost of 12 × 29 = 348 cents. This is a savings of 500 – 348 = 152 cents.

10. (any decimal between 2.01 and 2.33 or img)
It is given that: 1 < 3x – 5 < 2
Add 5 to each expression: 6 < 3x < 7
Divide each expression by 3: 2 < x < img
Grid in any decimal number or fraction between 2 and img = 2.33: 2.1, for example, or img which is the average of img and img

11. (8100) img

img

Since img must be an even integer, the greatest possible value of img is 18: img

12. (202) img After 33 repetitions of the pattern—red, white, white, blue, blue, blue—there will be 6 × 33 = 198 marbles in the box, of which 99 will be blue. When these are followed by 4 more marbles (1 red, 2 whites, and 1 blue), there will be 100 blue marbles, and a total of 198 + 4 = 202 marbles in all.

13. (28) Whether or not you can visualize (or draw) the second (large) square, you can calculate its area. The area of each of the four triangles is img for a total of 24, and the area of the 5 × 5 square is 25. Then, the area of the large square is 24 + 25 = 49. Each side of the square is 7, and the perimeter is 28.
img

14. imgimg

15. imgimg

16. (19) There are at most 14 blank cards, so at least 86 of the 100 cards have one or both of the letters A and C on them. If x is the number of cards with both letters on them, then img This is illustrated in the Venn diagram below.
img

17. (8190) img There are 26 × 26 × 9 = 6084 PIC’s with two letters and one digit, and there are 26 × 9 × 9 = 2106 PIC’s with one letter and two digits, for a total of 6084 + 2106 = 8190.

18. img If the diameter of the small white circle is d, then the diameter of the large white circle is 3d, and the diameter of the largest circle is d + 3d = 4d. Then the ratio of the diameters, and hence of the radii, of the three circles is 4:3:1. Assume the radii are 4, 3, and 1. Then the areas of the circles are 16img, 9img, and img. The sum of the areas of the white circles is 10img, the shaded region is 16img – 10img = 6img, and img

Section 8 Critical Reading

  1. (A).    Voles are similar to mice; however, they are also different from them, and so may be distinguished from them. Note how the use of “although” in the opening phrase sets up the basic contrast here.

(Contrast Signal)

  2. (C).    Because Dr. Drew’s method proved effective, it became a model for other systems. Remember to watch for signal words that link one part of the sentence to another. The “so...that” structure signals cause and effect.

(Cause and Effect Signal)

  3. (B).    The fact that the languages of the Mediterranean area were markedly (strikingly) alike eased or facilitated the movement of people and ideas from country to country. Note how the specific examples in the second part of the sentence clarify the idea stated in the first part.

(Examples)

  4. (E).    Feeling that a job has no point might well lead a person to perform it in a perfunctory (indifferent or mechanical) manner. Remember: watch for signal words that link one part of the sentence to another. “Because” in the opening clause is a cause signal.

(Cause and Effect Signal)

  5. (B).    Nelson remained calm; he was in control in spite of the panic of battle. In other words, he was imperturbable, not capable of being agitated or perturbed. Note how the phrase “in spite of ” signals the contrast between the subject’s calm and the surrounding panic.

(Contrast Signal)

  6. (E).    Despite his hard work trying to solve the problem, the solution was not the result or outcome of his labor. Instead, it was fortuitous or accidental. Remember to watch for signal words that link one part of the sentence to another. The use of the “was...and not...” structure sets up a contrast. The missing words must be antonyms or near-antonyms.

(Contrast Pattern)

  7. (E).    The italicized introduction states that the author has had his manuscript rejected by his publisher. He is consigning or committing it to a desk drawer to set it aside as unmarketable.

  8. (B).    The rejected author identifies with these baseball players, who constantly must face “failure.” He sees he is not alone in having to confront failure and move on.

  9. (B).    The author uses the jogger’s comment to make a point about the mental impact Henderson’s home run must have had on Moore. He reasons that, if each step a runner takes sends so many complex messages to the brain, then Henderson’s ninth-inning home run must have flooded Moore’s brain with messages, impressing its image indelibly in Moore’s mind.

10. (D).    The author is talking of the impact of Henderson’s home run on Moore. Registering in Moore’s mind, the home run made an impression on him.

11. (C).    The author looks on himself as someone who “to succeed at all...must perform at an extraordinary level of excellence.” This level of achievement, he maintains, is not demanded of accountants, plumbers, and insurance salesmen, and he seems to pride himself on belonging to a profession that requires excellence. Thus, his attitude to members of less demanding professions can best be described as superior.

12. (A).    The description of the writer defying his pain and extending himself irrationally to create a “masterpiece” despite the rejections of critics and publishers is a highly romantic one that elevates the writer as someone heroic in his or her accomplishments.

13. (C).    The author of Passage 2 discusses the advantages of his ability to concentrate. Clearly, he prizes his ability to focus on the task at hand.

14. (B).    When one football team is ahead of another by several touchdowns and there seems to be no way for the second team to catch up, the outcome of the game appears decided or settled.

15. (E).    The “larger point of view” focuses on what to most people is the big question: the outcome of the game. The author is indifferent to this larger point of view. Concentrating on his own performance, he is more concerned with the task at hand than with winning or losing the game.

16. (C).    Parade ground drill clearly does not entirely prepare a soldier for the reality of war. It does so only “to an extent.” By using this phrase, the author qualifies his statement, making it less absolute.

17. (C).    One would expect someone who dismisses or rejects most comparisons of athletics to art to avoid making such comparisons. The author, however, is making such a comparison. This reversal of what would have been expected is an instance of irony.

18. (C).    To learn to overcome failure, to learn to give one’s all in performance, to learn to focus on the work of the moment, to learn to have “the selfish intensity” that can block out the rest of the world—these are hard lessons that both athletes and artists learn.

19. (D).    Throughout Passage 2, the author stresses the advantages and the power of concentration. He believes that a person who focuses on the job at hand, rather than dwelling on past failures, will continue to function successfully. Thus, this author is not particularly swayed by the Passage 1 author’s contention that a failure such as giving up a key home run can destroy an athlete.

Section 9 Mathematical Reasoning

  1. (C).    Since the measure of a straight angle is 180°, x + y + 30 = 180. Replacing y by 2x, we get x + 2x + 30 = 180 ⇒ 3x = 150. So, x = 50 and y = 2x = 100.

  2. (C).    The temperature rose 8 – (–7) = 8 + 7 = 15° in 1.5 hours. The average hourly increase was 15° ÷ 1.5 = 10°.

  3. (D).    The expression n2 – 30 is negative whenever n2 < 30. This is true for all integers between –5 and 5 inclusive, 11 in all (including 0).

  4. (D).    The only thing to do is to test each set of values to see which ones work and which one doesn’t. In this case, choice D, a = 3 and b = –4, does not work: 2(3)2 + 3(–4) = 18 – 12 = 6, not 5. The other choices all work.

  5. (A).    The slope of the line, img, that passes through (–2, 2) and (3, 3) is img. The slope of any line perpendicular to img is img

  6. (C).    For some number x, the measures of the angles are x, 2x, and 3x; so img Therefore, the triangle is a 30-60-90 triangle, and the ratio of the sides is 1:img:2.

  7. (D).    By definition, a googol is equal to 10100. Therefore, g2 = 10100 × 10100 = 10200, which, when it is written out, is the digit 1 followed by 200 zeros, creating an integer with 201 digits.

  8. (E).    The graph of y = f(x – 3) is the graph of y = f(x) shifted 3 units to the right, as shown in Choice D. The graph of y = –f(x – 3) reflects Choice D in the x-axis, resulting in graph E.

  9. (A).    Since C = 2imgr, then img Since the formula for the area of a circle is A = imgr 2img

10. (B)imgimg

11. (C)imgimg

12. (E).    Joanna needed to drive the m miles in img hours. Since img, to find her rate, you divide the distance, m, by the time, img img

13. (A).    In the figure below, the area of img is img. Then the area of the shaded quadrilateral is 10 minus the areas of square ADEF [22 = 4] and triangle CDE [img(2)(2) = 2]: 10 – 4 – 2 = 4.

img

14. (D).    Since y is inversely proportional to x, there is a constant k such that xy = k. Then img Also, since y is directly proportional to z, there is a constant m such that img Then imgand img and so img

15. (E).    To find the average of three numbers, divide their sum by img To simplify this fraction, divide each term in the numerator by 3: img

16. (D).
img

By the Pythagorean theorem,

          img

and since the area is 20,

          img

Expand:

          img

Then

          img

Section 10 Writing Skills

  1. (D).    Error in logical comparison. Compare voices with voices, not voices with singers.

  2. (C).    Run-on sentence. Choice C corrects the error by turning the initial clause (“The … blackboard”) into a participial phrase (“After … blackboard”) and changing the subject of the main clause from he to the mathematics teacher.

  3. (C).    Error in usage. Do not use when after is in making a definition.

  4. (D).    Shift in number. The subject, students, is plural; the subject complement should be plural as well. Change tumbler to tumblers.

  5. (E).    Lack of parallelism. The “both … and” construction provides parallel structure.

  6. (B).    Wordiness. Choice B makes the writer’s point simply and concisely.

  7. (C).    Error in logical comparison. Compare audiences with audiences, not with theaters.

  8. (D).    Dangling participle. Ask yourself who is observing the preschoolers’ interactions.

  9. (C).    Error in choice of conjunction. Many people requested a deferment, but only a few received one. Use but rather than and to signal that what follows is contrary to what you might have expected.

10. (A).    Sentence is correct.

11. (A).    Sentence is correct.

12. (E).    Lack of parallelism. Choice E has parallel structure.

13. (C).    Error in subject–verb agreement. The subject, demand, is singular; the verb should be singular as well. Change are to is.

14. (D).    Sentence fragment. Choice D economically corrects the fragment.